Bar: MBE questions FOR REAL PROPERTY

Ace your homework & exams now with Quizwiz!

A nanny was caring for three children—ages three, five and seven—when her boyfriend called her to end their relationship because she was more devoted to the children than she was to him. To prove him wrong, the nanny left the three children by themselves for half an hour so she could talk to her boyfriend. She put all three children in a play area that was surrounded by a childproof gate. Although the seven-year-old knew how to open the gate, she gave her explicit instructions that all three kids must stay in the play area. While the nanny was gone, the three-year-old boy figured out how to open the childproof gate, and he headed straight for the kitchen. In the kitchen, he started to play with the stovetop knobs and inadvertently caused the gas to run. Soon, the children started to feel dizzy, nauseous, and confused. The seven-year-old went in search of her three-year-old brother and found him unconscious on the kitchen floor. She dragged him outside, but she was so dizzy that she was unable to carry her five-year-old brother outside. By the time the ambulance and fire department arrived, the five-year-old had already died from carbon monoxide poisoning. Can the nanny be convicted of involuntary manslaughter? Answers: A: Yes, because a gas leak occurred while the nanny left the children unsupervised. B: Yes, because the nanny's act was grossly negligent, and it put the children at a significant risk of serious bodily injury or death. C: No, because the nanny may have acted negligently, but she did not act recklessly. D: No, because the nanny did not have the intent to kill the children, and she did not commit an unlawful act.

Answer choice B is correct. Involuntary manslaughter is an unintentional homicide committed with criminal negligence or during an unlawful act. Criminal negligence is grossly negligent action that puts another person at a significant risk of serious bodily injury or death. It requires more than ordinary negligence for tort liability and something less than the extremely negligent conduct required for depraved-heart murder. Under the Model Penal Code rule, the defendant must have acted recklessly, which is a "gross deviation from the standard of conduct that a law-abiding person would observe in the actor's situation." Here, the nanny acted grossly negligently by leaving three children under the age of seven to fend for themselves so she could talk to her boyfriend. Answer choice A is incorrect because it implies a strict-liability standard, and this standard does not apply to involuntary manslaughter. Answer choice C incorrect because the nanny did behave recklessly. A law-abiding person tasked with caring for children would not leave three children under the age of seven by themselves. Answer choice D is incorrect because it is an incorrect statement of law with regards to involuntary manslaughter. The nanny does not need the intent to kill the children, nor does she need to commit an unlawful act in order to be guilty of involuntary manslaughter.

An amateur historian sought to purchase a house and its surrounding land because he believed, based on his own research, that the property had been the site of a key battle in the U.S. Civil War. The historian discussed this fact with the owner of the property as they inspected the land, telling the owner of his plans to dismantle the house and landscape the land to rent it to a local historical reenactment group who wanted to reenact the battle. The owner of the property had never heard of the battle, and although he listened with interest, he did nothing to expressly confirm or deny the historian's belief. The historian entered into a written contract with the owner for the property. A few days prior to the closing date, the historian contacted the historical reenactment group to offer the land as a site for a reenactment, but the group correctly explained to the historian that he was mistaken—the battle had actually taken place two miles north of his property. If the historian sues the owner to void the contract, is he likely to prevail? Answers: A: No, because the fact that the battle did not occur on the property did not materially affect the bargain. B: No, because the historian bore the risk of the mistake regarding where the Civil War battle actually took place. C: Yes, because the historian's purpose in entering into the contract was frustrated. D: Yes, because the owner's failure to confirm or deny the historian's belief constituted a breach of the duty of good faith.

Answer choice B is correct. When only one of the parties is mistaken as to an essential element of the contract, either party can generally enforce the contract on its terms. However, the mistaken party can void the contract if the he did not bear the risk of the mistake. Here, the historian bore the risk of the unilateral mistake regarding the actual location of the battle, because there is no indication that the risk of that mistake was assumed by the owner. Answer choice A is incorrect because, although the association of the property with Civil War history did not affect the condition of the property, the historian sought the property for its alleged historical significance, making it material to the bargain. Answer choice C is incorrect because the doctrine of frustration of purpose applies when unexpected events arise that destroy one party's basic assumption of the contract. Here, although the owner knew that the historian wanted to buy the land so that he could rent it out to a local historical reenactment group, his ability to do so was not a basic assumption of the contract. In addition, frustration of purpose requires that the frustration be so severe that it is not within the assumed risks inherent under the contract. Here, the possibility that the historian might not be able to rent the property to the reenactment group was within the assumed risks inherent to the contract. Answer choice D is incorrect because the duty of good faith generally does not impose an affirmative duty to act, but instead simply requires honesty in fact. Here, the owner did not create the historian's erroneous belief, and thus he was under no obligation to investigate or confirm the historian's claims.

A couple entered into a contract to purchase a house from the owner. The couple did not record the contract of sale. Prior to the execution of the contract, the owner incurred a debt to a creditor. Subsequent to the execution of the contract, the creditor obtained a judgment against the owner. Unaware of the contract of sale, the creditor recorded her judgment in the land records for the county in which the house was located, thereby giving the creditor a lien against property owned by the owner in the county. After the owner deeded the house to the couple and they recorded the deed, the creditor sought to execute the lien and levy on the house. The couple filed an action to enjoin the creditor from executing the lien. The applicable recording act reads: No conveyance or mortgage of real property shall be good against subsequent purchasers for value and without notice unless the same be recorded according to law. Who will prevail? Answers: A: The creditor, because she recorded her judgment prior the couple's recording of their deed and without notice of their purchase of the house. B: The creditor, because she had reduced her claim to judgment. C: The couple, because they were protected by the recording act as purchasers for value of the house. D: The couple, because the doctrine of equitable conversion protected their interest in the house from the judgment creditor.

Answer choice D is correct. Upon execution of the land sales contract, the couple became the equitable owners of the house; the owner merely held legal title which he was required to convey at closing to the couple. Consequently, the creditor's judgment lien, which was obtained after the contract was executed, was not enforceable against the house because the house no longer belonged to the owner. Answer choice A is incorrect because, although the creditor did record her interest in the property without notice of the sale of the property to the couple, the recording act does not protect a judgment creditor who is not deemed to be a purchaser for value. Answer choice B is incorrect because, while the creditor had reduced her claim to judgment, the judgment was against the owner. As such, the judgment was enforceable only against property owned by the owner. Once the contract was executed, the owner no longer was equitable owner of the house. Answer choice C is incorrect because the recording act does not protect the couple. They did not record the contract of sale and, although they did record the deed, they had constructive notice of the judgment lien because it had previously been recorded.

A man was drinking at a bar. He started arguing with the man sitting next to him, and the verbal argument quickly escalated into a physical altercation. The two men began punching each other, and then one of the men put on brass knuckles and started to hit the other man. The bartender quickly grabbed a knife from behind the bar, and tried to break up the fight between the two men. The man with the brass knuckles punched the bartender repeatedly. The bartender responded by trying to stab him. However, he accidentally stabbed a woman next to him instead. The woman has filed a battery claim against the bartender to recover damages for the stabbing. Who will prevail? Answers: A: The bartender, because he was trying to protect himself from the man. B: The bartender, because he did not intend to stab the woman with the knife. C: The woman, because the bartender's intent to stab the man transferred to her. D: The woman, because the bartender was required to retreat before using deadly force.

Answer choice A is correct. A person may use deadly force to defend himself if he has a reasonable belief that force sufficient to cause serious bodily injury or death is about to be intentionally inflicted upon him. In this case, the woman will not prevail because the bartender's use of deadly force, stabbing her with a knife, was reasonable to defend himself from the man's attack using brass knuckles. Answer choice B is incorrect because although transferred intent would usually apply to a misdirected battery, one who acts in self-defense is not liable for injuries to bystanders that occur while he is acting in self-defense, so long as those injuries were accidental, rather than deliberate, and the actor was not negligent with respect to the bystander. Answer choice C is incorrect because the doctrine of transferred intent is negated if the bartender used force necessary to defend himself from serious bodily injury or death. As stated above, one who acts in self-defense is not liable for accidental injuries to bystanders that occur while he is reasonably acting in self-defense. Answer choice D is incorrect. Under the majority rule, a person is not required to retreat before using deadly force.

A newly retired chemist bought a house in a small but densely populated rural town. The chemist still enjoyed conducting experiments in her spare time. Because she used very strong chemicals, she always experimented with her windows open to keep fresh air circulating in her home. A computer programmer, who lived a few doors down on the same road as the chemist, had lived in the neighborhood for 15 years. The programmer worked from home, and because he had many computers and other hardware that generated a lot of heat, he also kept his windows open to keep the house cooler. Since birth, the programmer had suffered from a rare disease known as anosmia, which prevented him from smelling anything. As a result, the programmer could not smell the noxious, though ultimately harmless, fumes emanating from the chemist's home, but he knew of them based upon community gossip. Setting aside the issue of damages, should the programmer prevail in a private nuisance action against the chemist? Answers: A: Yes, because other neighbors in the community were able to smell the fumes coming from the chemist's home. B: Yes, because the programmer suffered harm that is different from that suffered by other community members. C: No, because the programmer cannot smell the noxious odors emanating from the chemist's home. D: No, because he assumed the risk of encountering the malodorous smells by keeping his windows open.

Answer choice A is correct. A private nuisance is a thing or activity that substantially and unreasonably interferes with another individual's use or enjoyment of his land. The interference must be intentional, negligent, reckless, or the result of abnormally dangerous conduct to constitute nuisance. Finally, a substantial interference is one that would be offensive, inconvenient, or annoying to a normal, reasonable person in the community. The chemist's intentional experiments created smells that substantially and unreasonably interfered with the programmer's enjoyment of his home. Because it was annoying to other, presumably reasonable, neighbors in the community, the programmer can prevail as well under this theory. Answer choice B is incorrect because it states the standard for a private citizen's claim for public nuisance. Answer choice C is incorrect because it does not matter if the programmer cannot smell the fumes—it only matters if the average person would be offended, inconvenienced, or annoyed. However, the amount of damages the programmer recovers may be affected. Finally, answer choice D is not the best answer because although the programmer voluntarily and knowingly left his windows open even though he knew about the fumes, there was nothing inherently dangerous about leaving his windows open during the day so this would not be a strong defense to his recovery.

A foreign corporation owned cargo containers that were used in international commerce. These containers were registered and subject to taxation on the basis of their full value in the corporation's home country. A state imposed an ad valorem tax on cargo containers owned by the foreign corporation that were found within the state on the annual tax day, as the state did for all such cargo containers found within the state on that day. The number of such containers owned by the foreign corporation in the state on tax day was a fair representation of the number of such containers that could be found within the state on any given day throughout the year. The foreign corporation has challenged in federal court the imposition of this ad valorem tax on its cargo containers as a violation of the Commerce Clause. How is the court likely to rule on this challenge? Answers: A: For the foreign corporation, because the tax subjected the corporation to international multiple taxation on the cargo containers. B: For the foreign corporation, because the tax was imposed by a state on foreign commerce. C: For the state, because the state applied the tax in a nondiscriminatory manner to all containers found within the state on tax day. D: For the state, because the tax was fairly apportioned based on the number of cargo containers owned by the corporation that could be found within the state on any given day.

Answer choice A is correct. A state tax imposed on interstate commerce must satisfy the Complete Auto test. Under this test, (i) the activity taxed must have a substantial nexus to the taxing state, (ii) the tax must be fairly apportioned, (iii) the tax may not discriminate against interstate commerce, and (iv) the tax must be fairly related to the services provided by the state. A state tax on foreign commerce must, in addition to meeting the same requirements as a tax on interstate commerce, not (i) create a substantial risk of international multiple taxation or (ii) prevent the federal government from "speaking with one voice" regarding international trade or foreign affairs issues. Under these facts, because the cargo containers are subject to tax by the home country of the foreign corporation, the state tax subjects the containers to international multiple taxation, and thus the first of these two additional requirements has not been met. Consequently, the state tax on the cargo containers owned by the foreign corporation violates the Commerce Clause. Answer choice B is incorrect because a state is not prohibited by the Commerce Clause from taxing foreign commerce. However, in order to do so, the state tax must satisfy not only the Complete Auto test that applies to the taxation of interstate commerce but also two additional requirements imposed on state taxation of foreign commerce. Answer choice C is incorrect. Although the state ad valorem tax is applied in a nondiscriminatory manner and thereby satisfies one element of the Complete Auto test, this tax does not satisfy one of the additional requirements imposed on a state tax placed on foreign commerce. Answer choice D is incorrect because, although the state ad valorem tax was fairly apportioned and thereby satisfied the second element of the Complete Auto test, this tax does not satisfy one of the additional requirements imposed on a state tax placed on foreign commerce.

The vice principal at a public high school had suspicions that a particular 18-year-old student possessed a legally prescribed but controlled pain medication in violation of the school district rules. Although she did not have probable cause to conduct a search of the student, the vice principal believed she had a moderate chance of finding evidence of the student's possession of the medication. Pursuant to the vice principal's order, the student emptied his pockets in her office. Although the student did not have any pain medication in his pockets, among the items in the student's pockets were three tablets of an amphetamine that the student possessed illegally. The student was charged with possession of the amphetamine tablets. If the student moves to suppress the three tablets seized by the vice principal, should the court grant the student's motion? Answers: A: No, because the search was reasonable. B: No, because the student did not have a reasonable expectation of privacy with respect to items brought onto school grounds. C: Yes, because the vice principal lacked probable cause to search the student's pockets. D: Yes, because the vice principal was not searching for amphetamine tablets but for a pain medication that the student legally possessed.

Answer choice A is correct. Although the Fourth Amendment prohibits unreasonable searches and seizures by a government official and a warrantless search conducted without probable cause generally violates this prohibition, there is an exception for administrative searches, including searches of students by public school personnel. For such student searches to be reasonable, the school personnel must have a "moderate chance" of finding the expected evidence, and the measures adopted for the search must be reasonably related to the objectives of the search and not excessively intrusive in light of the age and sex of the student and the nature of the infraction. Here, these requirements appear to have been met. Therefore, the search was reasonable, and the evidence is admissible. Answer choice B is incorrect because a student retains a reasonable expectation of privacy in items brought onto school property. This expectation of privacy, however, must yield when school officials have reasonable grounds for a search of the student's belongings. Answer choice C is incorrect. Although probable cause is generally a touchstone for a search conducted by a governmental official to be valid, a public school official need only have reasonable grounds for conducting a search of a student. Answer choice D is incorrect. School personnel may search for an item that is prohibited from being in a student's possession pursuant to school rules, even though the student's possession of the item is legal. As with other governmental officials (e.g., the police), illegal items seized during a valid search are admissible, even though such items were not the reason for the search.

The owner of a coffee shop met a painter at an art show in April. Impressed with his work, the coffee shop owner asked the painter if he would like to paint a mural behind the counter at her new coffee shop. She told him that, due to construction, the mural could not be started until July 1 and would have to be completed before the grand opening on August 1. The painter told her that he was interested, but that he had applied for a competitive artist residency, and that he would be unable to do the work if he was accepted. They agreed to these terms. The following week, the painter signed a contract that provided that the painter would complete the mural by August 1 for $2,000. The contract, which stated that it was a full and final expression of their agreement, did not mention the residency restriction. On June 5, the painter was accepted to the residency, and he never began the mural. Is the coffee shop owner likely to recover for the painter's non-performance? Answers: A: No, because the contract was subject to a condition. B: No, because the painter's promise was illusory. C: Yes, because the painter agreed to perform a personal service involving taste and skill. D: Yes, because the parol evidence rule would bar the painter from presenting evidence of oral understandings not included in the final writing.

Answer choice A is correct. Although the written contract was a full integration of the owner's and painter's agreement, the contract is subject to the introduction of parol evidence that the existence of the contract was subject to a condition precedent. In this case, the artist is allowed to present evidence of his prior conversation with the coffee shop owner that the contract would not take effect if he were accepted to the residency. Answer choice B is incorrect because, although the condition precedent to the existence of the contract (the painter not being accepted to a residency program) rendered the contract ineffective, the painter did bind himself to paint the mutual if he was not accepted to the residency program. Answer choice C is incorrect because the nature of the painter's contractual duty does not obligate him to perform a duty discharged by the nonoccurrence of a condition precedent. Answer choice D is incorrect because parol evidence may be admitted to prove a condition precedent to the existence of the contract.

In order to protect valuable state resources from over-hunting, a state department enacted regulations establishing a lottery for permits to hunt elk in the state. The regulations put a cap on the percentage of permits granted to out-of-state residents. The regulations were challenged in court, and the federal appellate court found that the regulation violated the Dormant Commerce Clause and unconstitutionally discriminated against out-of-state commerce. In response to this decision, Congress passed legislation, which was signed by the President, that it was in the public interest for each state to continue to regulate the taking for any purpose of fish and wildlife within its borders, including by means of laws and regulations that differentiate between residents and nonresidents of the state. Subsequently, another state adopted a regulation that prohibited out-of-state commercial hunters from hunting elk within the state. A group of out-of-state residents who lead elk hunting expeditions sued the state official responsible for enforcing the regulation, seeking a declaratory judgment that the regulation violated the Dormant Commerce Clause. If the court determines that hunting is an economic activity that substantially affects interstate commerce, should the court grant the requested declaratory judgment? Answers: A: No, because Congress granted the states the right to discriminate against out-of-state residents. B: No, because hunting is a traditional area of state regulation. C: Yes, because the federal law improperly invades the province of the court. D: Yes, because the regulation discriminates against out-of-state residents.

Answer choice A is correct. Because Congress has exclusive authority over interstate commerce, it may explicitly permit states to act in ways that would otherwise violate the Dormant Commerce Clause. It must be unmistakably clear that Congress intended to permit the otherwise impermissible state regulation. Here, Congress has specifically provided that states may, through regulations as well as statutes, discriminate against out-of-state commercial hunters with regard to hunting and fishing within the state. Therefore, the regulation is permitted. Answer choice B is incorrect. Although hunting is an area that traditionally has been subject to regulation by the states, Congress may regulate economic activity that substantially affects interstate commerce. Consequently, the Tenth Amendment, which reserves to the states only those powers not delegated to the federal government or denied to the states, does not serve to permit this regulatory restriction on out-of-state hunters. Answer choice C is incorrect. Although the judiciary has the final say in interpreting the Constitution, here, Congress is engaged in its law-making function pursuant to the Constitution to regulate interstate commerce. Answer choice D is incorrect. Although the regulation clearly discriminates against out-of-state residents, Congress has expressly permitted this discrimination.

A manufacturer of t-shirts contracted with a new clothing store to sell the store 1,000 t-shirts per month for a period of two years. The clothing store's signature color for their clothing was an orange-tinted red color, called coquelicot, which is very difficult to replicate on a consistent basis. The contract specified that any t-shirts that were not coquelicot could be returned, but it was silent with regard to the return of any t-shirts for other reasons. One year into the contract, the store decided to switch to coquelicot-colored baseball hats instead of t-shirts. The store returned the most recent shipment of t-shirts to the manufacturer and demanded a refund. The manufacturer refused to grant the refund, and the store sued the manufacturer for damages. At trial, the manufacturer introduced the contract, which clearly stated that t-shirts that were not coquelicot could be returned. The store then attempted to introduce evidence that it had returned t-shirts for other reasons to the manufacturer in the past and received a refund. Is this evidence admissible? Answers: A: Yes, because the evidence is relevant to show that the manufacturer had accepted the return of coquelicot-colored t-shirts in the past. B: Yes, because the evidence can reasonably establish the parties' course of dealing on this issue. C: No, because the evidence regarding the return of the t-shirts violates the parol evidence rule. D: No, because the express term in the contract regarding the return of t-shirts takes precedence over the course-of-performance.

Answer choice A is correct. Course-of-performance evidence is admissible under the UCC to explain or supplement a contact. A course of performance is a sequence of conduct that is relevant to understanding an agreement between the parties if: (i) the agreement involves repeated occasions for performance by a party, and (ii) the other party accepts performance without objection and with knowledge of the course of performance. Here, there were monthly purchases of t-shirts and evidence that the manufacturer had accepted returns of coquelicot-colored t-shirts in the past. Therefore, this course-of-performance evidence is admissible. Answer choice B is incorrect because course of dealing refers to conduct between the parties based on another contractual relationship. Here, because there is only the one contract between the two parties and the evidence relates to the performance of that contract, rather than another contract, the evidence does not relate to their course of dealing, but rather their course of performance. Answer choice C is incorrect because the parol evidence rule does not prevent the introduction of evidence regarding conduct by the parties after the written contract was executed. Answer choice D is incorrect because the agreement was silent as to the return of coquelicot-colored t-shirts. Thus, course-of-performance evidence is admissible to explain how the parties viewed the right of the store to return such t-shirts.

During a family gathering, a farmer revealed that he had recently been diagnosed with a terminal illness. The farmer discussed the disposition of the family farm, which had been in the farmer's family for several generations, with his two adult children, a son and a daughter. Both children expressed an interest in retaining the farm in the family, although both lived elsewhere. However, the daughter, who had a medical practice, indicated that she had no interest in farming the land herself. The next day, the farmer called his attorney and instructed her to prepare a deed that immediately transferred the farm to his son. Later that week in the attorney's office, the farmer executed a valid deed in the presence of his attorney. In light of his medical condition, the attorney suggested that the farmer also make a will. The farmer indicated that he was uncertain, but that he thought that he had made a will when he was much younger. They made an appointment for the following week to review that will, if the farmer found it, and, if necessary, to draft a codicil or a will. As he was leaving, the attorney asked whether she should record the deed, the farmer instructed her to hold onto the deed and wait until after their next meeting so that he could consider everything at one time. The farmer did not discuss the existence or contents of the deed with anyone after he left his attorney's office. The next week, on his way to the appointment with his attorney, the farmer was killed in an automobile accident. In the farmer's car was found a will of which only the farmer was aware. The will, which had been drafted more than 20 years ago by another attorney now deceased, had been validly executed by the farmer. The will, which was made at a time during which the farmer had been estranged from his son, left the farm to his wife, who predeceased the farmer by 10 years, and named his daughter as the sole alternative beneficiary. After learning of the will, the daughter claimed ownership of the farm. In an appropriate action to determine ownership of the farm filed by the personal representative of the farmer's estate after admission of the will to probate, will the son be entitled to ownership of the farm? Answers: A: No, because the farm passes to the daughter under the terms of the will. B: No, because the will was executed before the deed. C: Yes, because the deed rather than the will governs ownership of the farm. D: Yes, because, since the deed was executed after the will, the deed superseded the will.

Answer choice A is correct. Even though the deed was validly executed, it did not operate to transfer the farm to the son because the delivery requirement was not satisfied. After execution of the deed, there was neither physical delivery of the deed to the son nor a recording of the deed. Instead, because the deed remained with his agent, the farmer retained the power to revoke it. Answer choice B is incorrect because the date of execution, while important in determining priority of conflicting wills, does not determine priority between a deed with inter vivos effect and a will. A will, regardless of when it is executed during the testator's lifetime, does not take effect until the testator's death. Answer choice C is incorrect because, as noted with respect to answer choice A, the delivery requirement was not satisfied. Consequently, the deed was not effective to transfer the farm out of the farmer's estate prior to his death. Answer choice D is incorrect because the fact that the deed was executed after the will does not give it priority over the will. In addition, as noted with regard to answer choice A, mere execution of a deed is not sufficient to transfer title; delivery is also required.

The holder of a patent for hybrid corn sued both an unlicensed wholesaler of that product and a retailer who purchased the corn from the wholesaler for alleged patent infringement. The patent holder reached an agreement with the retailer in which the holder released the retailer from liability in exchange for the payment of a nominal amount. After properly authenticating the agreement, the wholesaler sought to introduce it into evidence solely for the purpose of determining damages. The patent holder objected to the introduction of the agreement. Should the court admit the agreement into evidence? Answers: A: No, because the agreement constituted the acceptance of a settlement offer. B: No, because the danger of unfair prejudice outweighs the probative value of the agreement. C: Yes, because the agreement is being introduced solely for the purpose of determining the amount of damages. D: Yes, because the wholesaler was not a party to the agreement.

Answer choice A is correct. Evidence of a settlement offer, including evidence of the acceptance of such an offer, is not admissible for the purpose of establishing the validity of a claim or the amount of damages. For this reason, answer choice C is incorrect. Answer choice B is incorrect because it fails to correctly state the law. In order for evidence to be inadmissible under Federal Rule 403, the dangers of unfair prejudice must substantially outweigh its probative value. Answer choice D is incorrect because when there are more than two parties, a settlement agreement entered into by a party with an adverse party cannot be used by a remaining adverse party to prove or disprove the amount of an unsettled claim.

A plaintiff sued the driver of a car after the driver negligently crashed into the back of the plaintiff's car. Neither party suffered physical injury, nor were the cars damaged, but the defendant was concerned about the subsequent liability and was adamant that both parties remain until the police arrived to confirm that there had been no damage. They both spent an hour waiting for the police to arrive to take a report. The plaintiff was a musician who was on his way to a small venue for a performance, but he missed the performance due to the accident and subsequent waiting period. As it turned out, a talent scout had gone to the venue to see the plaintiff perform with the intent of signing him to a record label. Instead, the singer who replaced the musician for the performance was signed to a record deal. The plaintiff then sued the driver for the lost wages suffered from the missed performance, and he also sued for the wages he might have earned from the record deal. Can the plaintiff recover for the damages referenced above? Answers: A: He cannot recover for either the lost wages or the future loss. B: He can recover for lost wages only. C: He can recover for the future loss only. D: He can recover either the lost wages or the future loss, but not both.

Answer choice A is correct. The plaintiff must prove actual injury; nominal damages are not available in negligence actions, nor are damages for the threat of future harm. In addition, a plaintiff who suffers only economic loss without any related personal injury or property damage cannot recover such loss through a negligence action. Here, the plaintiff suffered only economic loss, and he cannot recover for that loss or the projected future losses. Answer choice B is incorrect because he cannot recover for pure economic loss if he faced no injury or property damage. Answer choice C is incorrect because he may not recover damages for the threat of future harm. Answer choice D is incorrect because he cannot recover for either.

A man believed that he and his wife were headed toward a divorce, and was concerned that his wife would fight to keep a valuable painting they had purchased together on their honeymoon. Although the man was staying at a hotel, he decided to break into the house and take the painting when he believed his wife would be out. So as not to draw any neighbors' attention by walking in the front door, the man climbed through an open window in the back. The wife was in fact home, and she was furious when she saw the man there. She pushed him repeatedly, telling him to get out of her house. The man pushed the wife back, and the wife hit her head on the corner of a table. Because she had a blood clotting disorder, the injury caused excessive bleeding in her brain, and she died the next day. The man has been charged with felony murder. The evidence at trial shows that the man knew of his wife's blood clotting disorder. Is the man likely to be convicted of felony murder? Answers: A: No, because the man did not commit an underlying felony. B: No, because the man did not intend to kill the wife. C: Yes, because the man knew of the wife's blood clotting disorder when he pushed her. D: Yes, because the wife was killed during the commission of an inherently dangerous felony.

Answer choice A is correct. Felony murder is an unintended killing proximately caused by and during the commission or attempted commission of an inherently dangerous felony, including burglary. To convict a defendant of felony murder, the prosecution must establish the underlying felony. In this case, the man did not commit a felony. The elements of burglary and robbery cannot be established because the man did not intend to take the property of another. Rather, he intended to take a painting that he owned. Because there was no felony, the man may not be convicted of felony murder. Answer choice B is incorrect because felony murder is by definition an unintentional killing; thus, the fact that the killing was unintentional would not prevent the man from being convicted of felony murder. Answer choice C is incorrect because it is irrelevant that the man knew of the wife's disorder for purposes of determining whether felony murder may be established. Answer choice D is incorrect because the man did not commit a felony.

An insurance company, incorporated and with its principal place of business in State A, brought a federal statutory interpleader action in federal district court after a dispute arose as to person(s) entitled under state law to the proceeds of an insurance policy in the amount of $100,000. The claimants under the policy are all citizens of State B. The insurance company posted a bond for $100,000, representing the disputed proceeds. Does the court have jurisdiction over the action? Answers: A: No, because the claimants are not diverse. B: No, because the claim does not arise under federal law. C: Yes, because the amount in controversy exceeds $75,000. D: Yes, because the insurance company is diverse from the claimants.

Answer choice A is correct. For statutory interpleader, diversity jurisdiction is met if any two claimants are citizens of different states. With regard to the amount in controversy, in a statutory interpleader action, the property at issue must merely be $500 or more in value, not meet the $75,000 threshold required for regular diversity matters. In this case, although the amount-in-controversy requirement is met, none of the claimants are diverse because they are all citizens of State B. Therefore, the court does not have jurisdiction over the statutory interpleader action brought by the insurance company. Answer choice B is incorrect. There is no requirement that a federal statutory interpleader action be based on federal law. Answer choice C is incorrect. The amount in controversy for a federal statutory interpleader action need only be $500 or more; it does not need to exceed $75,000. In addition, although the amount-in-controversy requirement is satisfied, none of the claimants are diverse. Thus, the court cannot exercise diversity jurisdiction over the action. Answer choice D is incorrect. Although the stakeholder (insurance company) must be diverse from the claimants in an action brought under the federal interpleader rule, the citizenship of the stakeholder is not relevant in an action for federal statutory interpleader.

A woman suffered from a debilitating disease, and her husband convinced her that having sexual intercourse with him would cure the disease. The husband knew that his statement was false. Relying on this statement, the woman gave her consent, and the two had sexual intercourse. Later, the woman learned that intercourse could not and did not cure her disease, and notified the police. Rape is statutorily defined as "sexual intercourse with a female against her will." The husband was convicted of rape. If the husband appeals the conviction, how should the appellate court rule on the appeal? Answers: A: Reverse the conviction, because the intercourse was not against the woman's will. B: Reverse the conviction, because the man was the married to the woman. C: Affirm the conviction, because the man obtained the woman's consent through fraudulent means. D: Affirm the conviction, because the man had the specific intent to rape the woman.

Answer choice A is correct. Fraudulent conduct does not negate consent in most situations. Here, the man induced the woman to consent to sexual intercourse through false promises--"fraud in the inducement"--but that, standing alone, does not negate consent under the common law. Here, the man made false promises to obtain consent, but did not conceal the actual nature of the act; consequently, the fraud was in the inducement, not in the factum. Answer choice B is an incorrect statement of the law; the applicable statute does not carve out immunity for persons accused of raping a spouse. Answer choice C is incorrect because, as discussed above, fraud in the inducement does not negate consent. Answer choice D is also an incorrect statement of the law; rape is a general intent crime.

A man and his friend decided to drive downstate to watch the homecoming football game at their alma mater. The man offered to drive and refused any compensation from the friend. On the way, the man negligently ran a stoplight and collided with another vehicle. The friend was severely injured. The friend sued the man for injuries sustained in the accident. The applicable jurisdiction recently enacted a guest statute with regard to automobile passengers. Will the friend prevail in her suit against the man? Answers: A: No, because the man was merely negligent. B: No, because a driver is liable only for injuries caused to passengers that have provided monetary compensation. C: Yes, because the man was strictly liable for injuries he caused to any passenger, including a guest. D: Yes, because the man owed his friend a duty to exercise ordinary care.

Answer choice A is correct. In most jurisdictions, an automobile driver owes a duty of ordinary reasonable care to all passengers, including guests (i.e., individuals who do not confer an economic benefit for the ride on the driver). However, a minority of states distinguish between the two with "guest statutes," which impose only a duty to refrain from gross or wanton and willful misconduct with respect to a guest. Here, the driver was merely negligent, and therefore the passenger would not be able to recover in a guest statute jurisdiction. Answer choice B is incorrect because a guest statute does not absolve the driver of liability to a non-paying passenger, but requires such passengers to establish a higher level of culpability by the driver in order to recover. Answer choice C is incorrect because a guest statute does not impose strict liability on the driver of a vehicle with respect to his passengers. Answer choice D is incorrect because, although most jurisdictions impose upon drivers a duty to exercise ordinary care with respect to guests as well as passengers, those that have guest statutes impose only a duty to refrain from gross or wanton and willful misconduct with a guest in the car.

A plaintiff properly filed a negligence action in federal district court based on diversity jurisdiction. Although either party could have filed a demand for a jury trial, neither party did. The case was heard before a judge. The defendant admitted liability, but contested the amount of damages sought by the plaintiff. The judge considered both documentary as well as testimonial evidence before detailing her findings and making the award. The defendant has timely appealed this award. What is the appropriate standard of review for the appellate court to apply to the amount of damages awarded by the trial judge? Answers: A: The award should not be set aside unless it is clearly erroneous. B: The award should be reviewed under an abuse of discretion standard. C: The preponderance of the evidence standard applies because this is a civil case. D: The appellate court should consider the evidence de novo because this is a bench trial.

Answer choice A is correct. The amount of damages to be awarded is a factual finding. As such, the award must not be set aside unless clearly erroneous. Answer choice B is incorrect because the abuse of discretion standard is typically applied to decisions made that relate to the conduct of the trial, such as the appropriate sanction to be imposed for the failure to comply with a discovery order. Answer choice C is incorrect. While the preponderance of the evidence standard is the typical evidentiary standard in a civil case for determining whether a fact has been proved, it is not the standard for appellate review of a trial judge's award of damages, which is a factual finding. Answer choice D is incorrect because the de novo standard of review applies to decisions made by the trial court as to the applicable law.

A jurisdiction defines receiving stolen property as (i) receiving control of stolen property, (ii) with the knowledge that the property is stolen, and (iii) with the intent to permanently deprive the owner of the property. A defendant, charged with receiving stolen property after the police found a stolen television in his home, denied that he knew it was stolen. On cross-examination, the prosecutor asked the defendant, "Didn't you also previously buy a stolen stereo from the same man who sold you this television?" The defendant's attorney immediately objected. What is the strongest basis for the defense attorney's objection? Answers: A: The probative value of the prosecutor's question is substantially outweighed by the danger of unfair prejudice. B: The prosecutor's question was irrelevant because it does not establish an element that the prosecutor must prove. C: The relevance of the prosecutor's question depends upon whether the defendant knew the stereo was stolen, and the prosecutor has not offered sufficient proof to support that finding. D: The risk of unfair prejudice is not substantially outweighed by the probative value of the prosecutor's question.

Answer choice A is correct. The general rule is that evidence must be relevant to be admissible, and all relevant evidence is admissible unless excluded by a specific rule or law. Relevant evidence can be excluded if its probative value is substantially outweighed by the danger of unfair prejudice, confusion of the issues, misleading the jury, undue delay, wasting time, or needlessly presenting cumulative evidence. Here, the probative value of the prosecutor's question regarding the alleged receipt of a stolen stereo in the past is likely substantially outweighed by the unfair prejudice that it will cause regarding the defendant's current crime—receipt of a stolen television. The prosecutor's question, and the defendant's potential answer, will only serve to prejudice the jury because the fact that the defendant may or may not have received a stolen stereo does not mean that he is guilty of receiving a stolen television in this instance. Answer choice B is incorrect because evidence need not, in and of itself, establish an element that a party must prove in order to be relevant. Under the test of relevancy, evidence is relevant even if it is only a single brick that is part of a wall of evidence establishing a party's position. Answer choice C is incorrect. When the relevance of evidence depends upon whether a fact exists, the court may admit the proposed evidence on the condition that the proof of its relevance is introduced later. Therefore, this is not the strongest basis for objecting to the question. Answer choice D is incorrect because it misstates the standard for excluding relevant evidence under Rule 403.

A salon owner contacted a manufacturer by email about purchasing shampoo sinks. The manufacturer sent the salon owner the following email: "I will sell you four shampoo sinks at a discounted price of $300 apiece." The salon owner responded immediately, rejecting the offer. However, due to a transmission problem in the internet routing system, the message was not delivered to the manufacturer until the following day. In the meantime, the salon owner contacted several other sellers, all of whom made significantly higher offers. The salon owner then sent another email to the manufacturer, stating, "I accept your offer." This email was delivered immediately. The following day, the misrouted rejection email arrived in the manufacturer's inbox. Assume the parties are in a jurisdiction that applies the mailbox rule to electronic communications. Was a contract formed? Answers: A: Yes, because the salon owner accepted the manufacturer's offer. B: Yes, because the mailbox rule applies. C: No, because the salon owner rejected the offer prior to accepting the offer. D: No, because the manufacturer received the salon owner's rejection.

Answer choice A is correct. The mailbox rule states that a timely sent acceptance is effective when sent, not upon receipt. However, if a communication is sent rejecting the offer, and a later communication is sent accepting the contract, the mailbox rule does not apply, and the first one to be received by the offeror prevails. Here, the salon owner rejected the offer, then sent an acceptance. Since the acceptance was received by the offeror first, the acceptance prevails. Answer choice B is incorrect because, as noted with respect to answer choice A, the mailbox rule does not apply when an acceptance is sent after a rejection. Answer choice C is incorrect because, while a rejection's effect is to terminate an offer, to be effective, the rejection must be communicated to the offeror. When a rejection is sent before an acceptance, the mailbox rule does not apply and the first communication received by the offeror prevails. Answer choice D is incorrect because a contract was formed when the manufacturer received the acceptance before the rejection. The fact that the manufacturer received the rejection the following day may affect the damages to which the manufacturer is entitled if the salon owner refuses to honor the contract, but it does not control the issue of the formation of a contract.

A carpenter agreed with a neighbor to construct a tree fort for the neighbor's child for $1,200. When the carpenter was halfway through the project, he abandoned the tree fort to work on more profitable projects. At that time, he had spent $300 on materials. When the carpenter approached the neighbor for payment, the neighbor refused. If the carpenter sues the neighbor for breach of contract, what is the likely amount of damages that the carpenter will be awarded? Answers: A: Nothing B: $300 C: $900 D: $1,200

Answer choice A is correct. The neighbor's payment of $1,200 to the carpenter was conditioned upon the carpenter's building a tree fort, and the carpenter's building a tree fort was conditioned upon the neighbor's payment of $1,200. As a contract for the performance of services, rather than for the sale of goods, the carpenter's substantial compliance with his obligation would have been sufficient to require the neighbor to pay him, although the neighbor would have been entitled to damages to the extent that the carpenter failed to fully complete the task. Here, however, the carpenter finished only one-half of the tree fort, and thus failed to substantially complete his contractual obligation. Consequently, the neighbor does not owe the carpenter any damages based on his breach of the contract. Answer choice B is incorrect because, as the breaching party, the carpenter is not entitled to recover his reliance damages of $300. Answer choice C is incorrect because, although the carpenter may be entitled to restitutionary recovery minus any damages the neighbor has suffered due to the breach, the carpenter, as the breaching party who has not substantially performed, is not entitled to recover damages based on a breach of the contract. Answer choice D is incorrect because the carpenter, as a breaching party who has not substantially performed, is not entitled to recover damages under the contract.

A plaintiff was on a crowded subway train during rush hour. The subway line was undergoing significant renovations, resulting in frequent, sudden stops by the subway trains. The plaintiff was standing in the middle of one of the subway cars and holding onto a pole for stability. The defendant, also standing in the subway car, was texting on his cell phone and not holding onto anything. The subway train came to a sudden stop causing the defendant to fall toward the plaintiff. The defendant lightly grabbed the plaintiff's arm to stop himself from falling completely over. The plaintiff did not like being touched by anyone as she had been in an abusive relationship in the past. Although she was not injured by the defendant's conduct, the plaintiff subsequently brought an action for battery against the defendant. Will the plaintiff prevail? Answers: A: No, because the plaintiff consented to the defendant's contact. B: No, because the plaintiff did not suffer any actual harm. C: Yes, because the defendant failed to exercise reasonable care. D: Yes, because the defendant intentionally grabbed her arm.

Answer choice A is correct. There is no battery if the plaintiff consented to the act, either expressly or by virtue of participating in a particular event or situation (such as being bumped on a crowded subway). In this case, the plaintiff chose to ride a crowded subway train. By making that decision, she implicitly consented to the ordinary contacts that can occur in that situation. The defendant lightly grabbing her arm to prevent himself from falling is within the type of contact that is typical on a crowded subway train. In addition, although the plaintiff found the contact offensive, a contact is only offensive when a person of ordinary sensibilities (i.e., a reasonable person) would find the contact offensive (objective test). Answer choice B is incorrect because proof of actual harm is not required to recover for battery. Answer choice C is incorrect. While the defendant's failure to exercise reasonable care would be relevant to a negligence claim, it is not relevant to a claim for battery. Answer choice D is incorrect. The fact that the defendant intentionally grabbed the plaintiff's arm to prevent his fall is not enough to impose liability for battery. Here, the contact was neither harmful nor offensive. In addition, the plaintiff impliedly consented to the contact.

A man who suffered from a mental illness shot and killed his neighbor after making plans to do so. The man was arrested and charged with murder. At trial, the man admitted that he intended to kill the neighbor, and that he appreciated that what he was doing was illegal. However, he also testified that he performed the killing under orders from his pet goldfish, who was possessed by a demonic spirit, and that he was unable to resist the goldfish's constant urging to kill the neighbor. The Model Penal Code test of criminal responsibility applies in the applicable jurisdiction. If the man timely and properly pleads that he was not criminally responsible, can he be found not guilty of murder but criminally responsible by reason of insanity? Answers: A: Yes, because the man was unable to resist the goldfish's constant urging. B: Yes, because the man suffered from a mental illness. C: No, because the man understood that killing his neighbor was illegal. D: No, because the man planned to kill his neighbor.

Answer choice A is correct. Under the Model Penal Code test, a defendant is not guilty when, at the time of the conduct, as a result of a mental disease or defect, he did not have substantial capacity to appreciate the wrongfulness of the act or to conform his conduct to the law. Here, although the man appreciated the wrongfulness of the killing, he was unable to resist the urging of the goldfish. Answer choice B is incorrect because the fact that the defendant suffered from a mental illness is not sufficient to establish an insanity defense. Answer choice C is incorrect because, as noted, the Model Penal Code test contains not only a cognitive prong but also a volitional prong; satisfying either is sufficient to establish an insanity defense. Answer choice D is incorrect because the volitional prong does not require that the defendant act impulsively, merely that he lacks the will to conform his conduct to the law.

A contractor decided to purchase a house to renovate and then sell. The contractor entered into a written contract with a homeowner for the purchase of the homeowner's house. The contract, signed by both parties, set forth the agreed upon price of $100,000 and a description of the property, as well as the terms of payment. No other information was included in the contract. After the contract was executed, the contractor received notice that another house he was interested in renovating had become available to purchase for $65,000. Realizing that he would be able to obtain a significantly higher profit from the newly available house, the contractor called the homeowner to inform him that he was backing out of their contract and would not purchase the house. When the homeowner stated that he would try to enforce the contract in court, the contractor argued that the contract was unenforceable because it did not set forth the closing date. The homeowner subsequently brought an action against the contractor for specific performance of the contract. Will the homeowner prevail? Answers: A: Yes, because the contract is enforceable under the Statute of Frauds. B: Yes, because the homeowner detrimentally relied on the contract. C: No, because a seller cannot seek specific performance of a land sales contract. D: No, because the contract did not contain all of the essential terms.

Answer choice A is correct. Under the Statute of Frauds, a land sales contract must (i) be in writing, (ii) be signed by the party to be charged, and (iii) contain all of the essential terms (i.e., parties, property description, terms of price and payment). Here, the facts state that the contractor entered into a written contract with the homeowner. The contract was signed by both parties and it included the price, a description of the property, and the terms of payment. Therefore, the contract satisfied the Statute of Frauds and can be enforced against the contractor. Answer choice B is incorrect. Specific performance may be permitted when the party seeking enforcement has reasonably relied on the contract and would suffer such hardship that the other party will be estopped from asserting the Statute of Frauds as a defense to the contract. Here, there are no facts that indicate the homeowner detrimentally relied on the contract with the contractor. Moreover, the contractor cannot raise the Statute of Frauds as a defense here because its requirements have been satisfied. Answer choice C is incorrect. Under the theory of mutualities of remedies, the seller (the homeowner) is permitted to seek specific performance and force the buyer to purchase the land, even though the seller typically receives money rather than land. Answer choice D is incorrect. The closing date, the date on which performance will occur under the contract, is not considered an essential element that must be in writing under the Statute of Frauds. If no closing date is specified in the contract, the court will infer a reasonable time from the date of the contract.

A woman placed an online order to purchase a unique tea kettle from a kettle manufacturer as a gift for her brother. The kettle was square-shaped and had two spouts. The kettle's shipping box stated the following: "Caution! The enclosed kettle is a novelty item and should not be used to pour boiling water. Use with boiling water may result in steam burns." Before using the kettle, the brother read some online reviews of the kettle. Many reliable reviews stated that steam burns were very common when using the kettle, because steam would escape from the second spout if the kettle was not held properly. However, many reviews suggested that the kettle could be used safely if it was held in a particular way. The first time the brother used the kettle, he carefully followed the instructions from the reviews on how to hold the kettle when filled with boiling water. However, while he was pouring the boiling water from one spout, scorching steam escaped from the other spout and burned the brother's skin. The jurisdiction applies the common-law rules for contributory negligence and assumption of the risk. If the brother files a strict-products-liability suit against the manufacturer, what is the manufacturer's best defense? Answers: A: The brother assumed the risk of being burned. B: The brother did not use the kettle properly. C: The brother received the kettle as a gift. D: The brother was negligent in using the kettle with boiling water.

Answer choice A is correct. Under the doctrine of strict products liability, a seller of a product is liable for personal injuries caused by that product, even in the absence of fault, if the product was defective, the defect existed at the time the product left the defendant's control, and the defect caused the plaintiff's injury when used in an intended or reasonably foreseeable way. However, a voluntary and knowing assumption of the risk is a complete bar to recovery in contributory-negligence jurisdictions if a plaintiff is aware of the danger and knowingly exposes himself to it. Here, the brother knew that steam burns were possible if the novelty kettle was used with boiling water, but decided to use the kettle with boiling water anyway. Accordingly, assumption of the risk provides the manufacturer with the strongest defense to the brother's strict-products-liability suit. Answer choice B is incorrect. Mere misuse will not constitute a defense to a strict products liability claim if the misuse is reasonably foreseeable. Here, the fact that the brother misused the kettle would likely be a foreseeable misuse of the kettle, especially based on the number of online reviews indicating users had used the kettle with boiling water. Therefore, this argument would not protect the manufacturer from liability. Answer choice C is incorrect. Anyone foreseeably injured by a defective product may bring a strict-liability action. Appropriate plaintiffs include not only purchasers, but also other users of the product. There is no requirement that the brother must be a purchaser in order to recover. Answer choice D is incorrect. In a contributory-negligence jurisdiction, the plaintiff's negligence generally is not a defense to a strict-products-liability action when the plaintiff negligently failed to discover the defect or misused the product in a reasonably foreseeable way. Ordinary contributory negligence by the plaintiff will not bar recovery based on strict products liability. Therefore, the manufacturer will only have a chance to succeed in defending against this action if it argues that the brother assumed the risk of steam burns.

A son and a daughter are opposing parties in federal court. At trial, the daughter presented evidence that her father has been missing for ten years, and that no one has heard from him in that time. The son testified that he received a phone call three years ago from a person that he believes was his father. In the jurisdiction, a rebuttable presumption arises that a person is dead when a party establishes that the person has been missing and not heard from for more than seven years. Which of the following is correct? Answers: A: The jury must find that the father is dead. B: The jury may find that the father is dead. C: The burden has shifted to the son to persuade the jury that the father is alive. D: The judge must instruct the jury to conclude that the father is dead.

Answer choice B is correct. A presumption is a conclusion that the trier of fact is required to draw upon a party's proof of an underlying fact or set of facts (i.e., basic facts). A rebuttable presumption shifts the burden of production, but not the burden of persuasion, to the opposing party. However, a rebuttable presumption may be overcome by evidence to the contrary. If no contrary evidence is introduced, the judge must instruct the jury to accept the presumption. If contrary evidence is introduced, as is the case here, then the presumption no longer has a preclusive effect. At this point, the jury may, but is not required to, draw the conclusion from the basic facts. Thus, the jury may determine the weight and credibility of all of the evidence. For this reason, answer choice A is incorrect. Answer choice C is incorrect because a rebuttable presumption shifts the burden of production, but not the burden of persuasion, to the opposing party. Answer choice D is incorrect because it is an incorrect statement of law. After a rebuttable presumption no longer has a preclusive effect due to the introduction of contrary evidence, a judge may instruct the jury that it may, but is not required to, draw the conclusion from the basic facts.

A private high school was in the market for new desks and chairs for its classrooms. It had inquired into the cost of acquiring 1,000 new desks and chairs from a particular vendor. On June 15, the vendor sent a signed letter to the private high school offering to sell 1,000 desks and chairs for $30,000. The letter stated that an acceptance of the offer by the high school would only be effective if it was received by June 25. On June 23, the private high school mailed its signed, written acceptance of the vendor's offer. On June 24, the vendor, after concluding that the price it originally requested was too low, directly notified the private high school that it was revoking its offer. The vendor received the private high school's acceptance on June 26. Was an enforceable contract between the private high school and the vendor formed? Answers: A: No, because the private high school's acceptance was received on June 26. B: No, because the vendor revoked the offer on June 24. C: Yes, because the private high school mailed its acceptance on June 23. D: Yes, because the vendor's offer was irrevocable.

Answer choice A is correct. Under the mailbox rule, an acceptance that is mailed within the allotted response time is effective when sent (not upon receipt), unless the offer provides otherwise. Here, the vendor's offer stated that an acceptance would only be effective if it was received by June 25. Thus, the mailbox rule does not apply. The vendor received the private high school's acceptance on June 26. The acceptance is ineffective because it was not received by June 25. Therefore, the offer was no longer available when the acceptance was received. As a result, no contract was formed. Answer choice B is incorrect. Under the UCC, an offer to buy or sell goods is irrevocable if the offeror is a merchant, there is assurance that the offer is to remain open, and the assurance is contained in a signed writing by the merchant. Here, the vendor was a merchant and he informed the private high school in a signed letter that it could accept his offer by June 25. Accordingly, the vendor's offer was irrevocable until June 25. Thus, the vendor's attempted revocation of the offer on June 24 was ineffective. Answer choice C is incorrect because the acceptance had to be received by June 25—it was not effective upon mailing. Answer choice D is incorrect. Although the vendor's offer was irrevocable under the UCC firm offer rule, the offer was only irrevocable until June 25. The offer was no longer available on June 26 when the vendor received the private high school's acceptance.

BabyCo, a manufacturer of baby gear, designed a baby bouncer that allowed infants to jump up and down without parental assistance. The baby bouncer could be attached to a doorframe with a plastic clamp, which was connected to the baby seat via a spring and safety cord. BabyCo designed the baby bouncer to safely withstand a maximum weight of 20 pounds, but if an infant exceeded that weight, there was a risk that the clamp could slip off of the doorframe and severely injure the infant. For a slightly higher cost, BabyCo could have used a metal clamp that withstood a maximum weight of 40 pounds, but they opted for the cheaper plastic clamp so that they could undercut their competitors. BabyCo attached a warning label to the bouncer stating the weight limit and that if the limit was exceeded, the clamp could slip off the doorframe and injure the infant. A mother purchased a baby bouncer manufactured according to BabyCo's specifications for her nine-month-old infant who weighed 25 pounds. She read the warning label but chose to ignore the weight limit. After a few uses, the clamp slipped off the doorframe, and the infant suffered a concussion and broken leg from the fall. The mother brought a strict products liability action against BabyCo. The jurisdiction is a traditional contributory-negligence jurisdiction. Who will prevail? Answers: A: BabyCo, because the mother chose to ignore the weight limit on the warning label. B: BabyCo, because they put a warning label on the baby bouncer regarding the weight limit. C: The mother, because the weight limit was the result of a manufacturing defect. D: The mother, because the baby bouncer manufactured by BabyCo was defectively designed.

Answer choice A is correct. Voluntary and knowing assumption of the risk is a complete bar to recovery in contributory-negligence jurisdictions. Assumption of the risk is a subjective standard. The plaintiff must be aware of the danger and knowingly expose himself to it. In this case, BabyCo attached a warning label to the bouncer stating the weight limit and that if the limit was exceeded, the clamp could slip off the doorframe and injure the infant. Despite reading this warning, the mother chose to ignore it. In placing her baby who exceeded the weight limit in the bouncer, the mother voluntary and knowingly assumed the risk that the clamp could slip off the doorframe. Therefore, the mother likely cannot recover in this traditional contributory-negligence jurisdiction. Answer choice B is incorrect because BabyCo cannot generally escape liability by placing a warning label on the baby bouncer. If this were permitted, then BabyCo could escape liability for a defectively designed baby bouncer by simply putting a label on it. The presence of the warning here, however, is relevant to the mother's knowledge of the risk. Answer choice C is incorrect because the baby bouncer was manufactured according to the proper design specifications, so there was no manufacturing defect involved. Answer choice D is incorrect. Under strict liability, the manufacturer of a defective product may be liable for any harm to persons or property caused by such product. A product is defective when, at the time of the sale or distribution, it contains a design defect—which is the case here. Courts apply the risk-utility test when determining if a design defect exists, and analyze whether the risks posed by a product outweigh its benefits. The plaintiff must prove that a reasonable alternative design that was economically feasible was available to the defendant, and the failure to use that design rendered the product unreasonably safe. Here, BabyCo could have used metal clamps for a slightly higher cost, and their failure to incorporate them into the design rendered the baby bouncer unreasonably unsafe. However, the mother's decision to place her baby in the bouncer with full knowledge of the product defect will result in BabyCo prevailing in the lawsuit.

The owner of pastureland permits a herder to keep several goats in the pasture. The goats wander off the land, leap over a fence properly maintained by a neighbor, and ramble into the neighbor's garden. Once in the garden, the goats eat the vegetables growing there. In a strict liability action brought by the neighbor against the landowner and the herder, who is liable? Answers: A. Only the landowner. B. Only the herder. C. Both the landowner and the herder. D. Neither the landowner nor the herder.

Answer choice B is correct because the owner of any animal, wild or domestic (other than a household pets) is strictly liable for any reasonably foreseeable damages caused by the animal while trespassing on another's land. Strict liability does not extend to the owner of the land on which the animals are kept, even when the animals are on the land with the landowner's permission, unless the landowner also has the right to possess the animals. Consequently, answer choices A, C, and D are incorrect.

A wealthy mother disapproved of her daughter's boyfriend, a mechanic in a small auto shop. Despite her protestations, the daughter refused to break up with him, and the mother decided to get rid of the boyfriend before it was too late. She told her husband that she wanted the boyfriend dead. The husband formulated a plan to kill the boyfriend but to make it look like an accident by sneaking into the auto shop in the middle of the night and rigging a car that the boyfriend was working on to fall on top of him; he snuck into the shop and did so later that night. The next morning, however, the mother changed her mind and told her husband not to go through with the plan. Neither the husband nor the wife went to the auto shop to make the car safe, nor did they warn the boyfriend that there was any danger awaiting him at the auto shop. The boyfriend went to work that day, and the rigged car fell on him and crushed him to death. The husband was arrested for first-degree murder due to a hidden security camera in the garage. He subsequently exposed his wife's participation in the plan, and she was arrested as well. A state statute defines first-degree murder as premeditated and deliberate. The jurisdiction follows the Model Penal Code. What is the most serious crime, listed below in descending order of seriousness, for which the mother can be convicted? Answers: A: First-degree murder B: Conspiracy C: Solicitation D: No crime

Answer choice B is correct. A conspiracy is an agreement to accomplish an unlawful purpose plus the intent to accomplish that purpose. The Model Penal Code (MPC) and the majority rule require a legal or illegal overt act in furtherance of the conspiracy. Here, the husband's rigging of the car to fall on top of the boyfriend is clearly an overt act in furtherance of the conspiracy. The MPC allows subsequent withdrawal by a co-conspirator after the overt act, but the defendant must thwart the performance of the planned crimes. The wife told the husband that she no longer wanted to go through with killing the boyfriend, but she did not thwart the plan so she is guilty of conspiracy under the MPC. Answer choice A is incorrect because although the wife was unable to effectively withdraw from the conspiracy, her withdrawal was effective with respect to the substantive crime of first-degree murder committed by her husband. Answer choice C is incorrect because although the wife is guilty of solicitation, she is also guilty of conspiracy, which is a more serious crime. Answer choice D is wrong because the wife is guilty of both solicitation and conspiracy.

An attorney was a sole practitioner specializing in family law. Her niece was a recent law school graduate, and her nephew was an attorney. The attorney decided to retire, and conveyed the historic building that housed her law practice "to my niece, but if she fails to pass the bar exam within a year of her law school graduation, to my nephew." Which of the following is an accurate description of the property interests created? Answers: A: The niece has a fee simple subject to condition subsequent, the nephew has a right of re-entry, and the attorney has no interest. B: The niece has a fee simple subject to an executory interest, the nephew has an executory interest, and the attorney has no interest. C: The niece has a fee simple determinable, and the attorney and the nephew each have a possibility of reverter. D: The niece has a fee simple subject to condition subsequent, the attorney has a right of re-entry, and the nephew has an executory interest.

Answer choice B is correct. A fee simple subject to an executory interest is a present fee simple estate that is limited in duration by either conditional language or durational language, such that it will terminate upon the occurrence of the specified condition, and title will pass to a third party (i.e., someone other than the grantor). Upon occurrence of the stated condition, the present fee simple terminates automatically. The future interest held by the third party is an executory interest. In this case, the niece had a fee simple subject to an executory interest due to the conditional language in the grant. The nephew had an executory interest that would become a possessory interest if the niece did not pass the bar exam. Answer choices A and D are incorrect because the title would pass to a third party, rather than the grantor. A fee simple subject to a condition subsequent is a present fee simple that is limited in duration by specific conditional language, such as "provided that." With a fee simple subject to condition subsequent, the grantor may terminate the estate upon occurrence of the stated condition. Here, the grantor did not retain the right to terminate the estate, but rather gave the nephew an executory interest. Answer choice C is incorrect. A fee simple determinable is a present fee simple estate that is limited by specific durational language (e.g., "so long as," "while," "during," "until"), such that it terminates automatically upon the happening of a stated condition, and full ownership of the property is returned to the grantor. In this case, the ownership did not return to the grantor upon the occurrence of the stated condition.

A tenant rented a two-story building; she operated a store on the first floor of the building and lived on the second floor. During a winter weekend when the tenant was out of town and the store was closed, a thief broke into the building. The thief left the building exposed to the elements. As a consequence, water in the pipes froze and burst the pipes, resulting in structural damage to the building. When the tenant returned and discovered the problem, she promptly notified the landlord. The lease contains a provision obligating the tenant to maintain and repair the premises during the tenancy. Which of the following would be the tenant's best argument that she is not obligated to repair the damage to the building? Answers: A: The criminal act of a third party caused the damage. B: The lease was residential. C: The structure of the building was damaged. D: The damage was due to natural causes.

Answer choice B is correct. A residential lease generally cannot place the duty to make repairs on the tenant, and a provision to that effect is void. This would certainly be true if the need for the repairs did not arise from the tenant's acts and the tenant promptly notified the landlord of the need for such repairs, as is the case here. Answer choice A is incorrect because, even if the damage is attributable to the criminal act of a third party, a provision in a commercial lease that the tenant is obligated to maintain and repair the premises may still impose the duty to repair on the tenant even in this instance. Answer choice C is incorrect because, although it is possible that damage affecting the structure of the building may defeat a provision in a commercial lease obligating the tenant to repair the premises, because the duty to repair is less likely to be imposed in a residential lease, the tenant's best argument is that she is a resident of the premises. Answer choice D is incorrect because, even if the damage to the premises is attributable to natural causes, if the provision is in a commercial lease, the duty to repair may still be placed on the tenant.

A defendant was charged with possession of cocaine with intent to distribute. Before trial, the defendant attended a meeting with his attorney and the prosecutor where they discussed the defendant's possible cooperation with the government. During the meeting, the defendant revealed incriminating information about his involvement with a gang that distributed cocaine. No plea deal was reached. At trial, the defendant testified that he had never possessed cocaine. To impeach the defendant's testimony, the prosecutor sought to question the defendant about the incriminating statements he made during the pre-trial meeting. The defendant's attorney objected. Should the court permit such questioning? Answers: A: No, because a defendant who testifies is not subject to impeachment by a prior inconsistent statement. B: No, because the statements were made during plea negotiations. C: Yes, because no plea deal was ever reached between the defendant and the prosecutor. D: Yes, because the incriminating statements were being introduced for impeachment purposes.

Answer choice B is correct. A statement made by a defendant while negotiating a plea is generally not admissible as evidence against the defendant. Answer choice A is incorrect because a defendant who testifies is subject, like any other witness, to impeachment by a prior inconsistent statement. However, since the defendant's statements were made during an unsuccessful plea negotiation, such statements are generally not admissible as evidence. Answer choice C is incorrect because the exclusion of a statement made during plea negotiations specifically applies to protect the defendant when a plea is not reached. Answer choice D is incorrect because the exclusion applies when the defendant's statement is being used for impeachment as well as substantive purposes.

An indigent defendant was arrested and charged with burglary. At his preliminary hearing, counsel was appointed. Unable to satisfy the conditions for his pre-trial release, the defendant was detained in jail awaiting trial on the burglary charge. While the defendant was in jail, a police detective questioned him about an unrelated murder charge without his lawyer present. Prior to questioning, the police detective revealed that she was a police officer and Mirandized the defendant. The defendant subsequently made an incriminating statement with regard to the murder, which lead to him being charged for that crime as well. At his trial for the murder, the defendant timely moved to suppress the statement he made to the police detective as having been obtained in violation of his constitutional rights. Should the court grant this request? Answers: A: No, because the defendant was in custody for an unrelated burglary. B: No, because the defendant was given Miranda warnings before making his incriminating statement. C: Yes, because the defendant's Fifth Amendment right to counsel was violated. D: Yes, because the defendant's Sixth Amendment right to counsel was violated.

Answer choice B is correct. Although the defendant had a Fifth Amendment right to counsel with regard to the unrelated murder because he was in custody and being interrogated about the crime, he had been informed of his Miranda rights before making the incriminating statement. Absent an assertion of his right to counsel prior to making the statement, the defendant cannot now assert that his Fifth Amendment right to counsel was violated. Answer choice A is incorrect. Once the Sixth Amendment right to counsel attaches, it applies only to the specific offense at issue in those proceedings. Here, the defendant's Sixth Amendment right to counsel only applied to the burglary charge. The defendant's Fifth Amendment right to counsel is triggered only when the defendant is in custody. However, the reason the defendant is in custody is irrelevant to his Fifth Amendment right to counsel. Therefore, the fact that he was in custody for an unrelated offense is not determinative of his right to suppress the evidence. Answer choice C is incorrect. Although the defendant had a Fifth Amendment right to counsel with regard to the uncharged murder, he had been informed of his Miranda rights before making the incriminating statement. Therefore, he waived this right to counsel by making the statement. Answer choice D is incorrect. Although the defendant's Sixth Amendment right to counsel prevents the defendant from being questioned about the burglary, this right is offense-specific and only attaches once the defendant has been indicted or formally charged with a crime. Therefore, the defendant's Sixth Amendment right to counsel was not violated when he was questioned about the unrelated murder.

An off-duty police officer was having a drink with his wife in a bar. A man walked by their table, and made a lewd comment about the wife as he passed. The officer jumped to his feet and asked the man to repeat what he had said. The man, who was obviously intoxicated, repeated the comment. The officer then pushed the man, causing the man to fall backwards. The man rose to his feet, pulled a knife from his waistband, and walked toward the officer. As the man lunged at the officer with the knife, the officer pulled his gun from his waistband and shot the man in the leg. Although the officer did not intend to fire a lethal shot, the defendant had a disease that prevented his blood from properly clotting, and he died from blood loss due to the injury. Is the officer likely to be convicted of homicide? Answers: A:No, because the officer had no knowledge of the man's disease when he shot him. B: No, because the officer was entitled to use self-defense under the circumstances. C: Yes, because the officer did not communicate his intent to withdraw from the altercation. D: Yes, because the officer intended to do grievous bodily harm.

Answer choice B is correct. An initial aggressor gains the right to act in self-defense when an aggressor using nondeadly force is met with deadly force or the aggressor, in good faith, completely withdraws from the altercation and communicates this fact to the victim. In this case, the officer's nondeadly force was met with deadly force, and thus he was entitled to use self-defense. Answer choice A is incorrect because, if the officer had not been justified in using self-defense, he could be held liable for the death of the man because he intended to do grievous bodily injury to the man. Answer choice C is incorrect because the officer did not need to communicate his intent to withdraw in this instance because his use of nondeadly force was met with deadly force. Accordingly, the officer was entitled to use self-defense. Answer choice D is incorrect because the officer was entitled to use self-defense and thus would not be held liable for the man's death despite his intent.

A father was throwing a wedding for his daughter, and the wedding costs were adding up so quickly that the father was forced to take out a second mortgage on his home, as well as a bank loan at a very high interest rate. The father was under a great amount of financial pressure, but he was willing to do anything to give his daughter her dream wedding. After many months of stress and anxiety, his daughter's wedding day finally arrived. While waiting in the buffet line to get some dinner at the wedding reception, the father overheard his daughter's mother-in-law tell another guest that she thought the wedding was low-rent and tacky, and that her daughter-in-law's father was a penny-pinching, tightfisted man. The father became enraged and could not control himself. He grabbed a knife from the roast beef carving station and stabbed his daughter's mother-in-law to death. A state statute defines murder in the first degree as premeditated and deliberate, murder in the second degree as common-law murder, and voluntary manslaughter under the common-law rule. What crime did the father commit? Answers: A: First-degree murder B: Second-degree murder C: Voluntary manslaughter D: Involuntary manslaughter

Answer choice B is correct. Common-law murder is the unlawful killing of another human being committed with malice aforethought. Malice aforethought includes the following mental states: intent to kill, intent to do serious bodily injury, reckless indifference to an unjustifiably high risk to human life (depraved heart), or intent to commit certain felonies (felony murder). Here, the father intended to do serious bodily injury when he stabbed the mother-in-law with a carving knife. He also demonstrated a callous disregard for human life when he stabbed her over some disparaging comments about the wedding and his alleged tight-fistedness. Answer choice A is incorrect. Based upon the facts, the father's actions were not premeditated, because there is no indication that the father planned to kill the mother-in-law in advance. Answer choice C is incorrect. For a defendant to be found guilty of voluntary manslaughter, there must be adequate provocation arousing a sudden or intense passion in the mind of an ordinary person. The defendant must suffer a loss of control, must not have sufficient time to cool off, and must in fact not have regained her self-control before the killing of the victim. Here, the father suffered a loss of control when he overheard the mother-in-law, and he did not regain self-control before he stabbed her to death. However, words alone are not adequate, no matter how provocative. Answer choice D is incorrect because the homicide was not committed with criminal negligence or during an unlawful act; it was an intentional act.

A homebuyer borrowed money to finance her purchase of a residence. The homebuyer executed a promissory note, payable to the lender, in the amount of the loan plus interest and gave the lender a mortgage on the residence as security for repayment of the loan. Subsequently, the homebuyer sold the residence to a friend. As part of the sales agreement between the homebuyer and her friend, the friend assumed liability for payment of the note. Later, when the value of the home exceeded the outstanding obligation on the note, the lender released its mortgage interest in the home. Shortly thereafter, the loan fell into default. Can the lender successfully maintain an action against the original homebuyer for repayment of the loan? Answers: A: No, because the friend assumed personal liability for payment of the note. B: No, because the lender released its mortgage interest in the home. C: Yes, because the homebuyer executed a promissory note, which was payable to the lender. D: Yes, because, upon repayment of the loan, the original homebuyer is subrogated to the lender's rights against the friend with respect to the note.

Answer choice B is correct. Despite the homebuyer's sale of the residence to a friend and the friend's assumption of liability on the note, the homebuyer remained liable on the note. By releasing the mortgage, the lender has eliminated the homebuyer's recourse to the mortgage to recover, at least to the extent of the value of the residence, the amount that the homebuyer is obligated to pay the lender pursuant to the note. Consequently, the homebuyer's obligation is discharged as a result of the lender's release of the mortgage. This is true even though the homebuyer also has recourse to seek the personal liability of her friend because of the assumption agreement between them. Answer choice A is incorrect. Even though the friend did assume personal liability for payment of the note, this assumption agreement, through which the friend became primarily liable for payment of the note and the homebuyer secondarily liable as a surety, was only between the homebuyer and her friend. Consequently, the lender can maintain an action to enforce the note against the homebuyer without first seeking recovery from the friend. However, such an action will not be successful because the lender, by releasing the mortgage, has eliminated the homebuyer's recourse to the mortgage. Answer choice C is incorrect because, even though the lender can seek to enforce the note against the homebuyer, such an action will not be successful as the lender has destroyed the homebuyer's recourse to the mortgage. Answer choice D is incorrect because, even though the homebuyer would be subrogated to the lender's rights against the friend with respect to the note if the homebuyer repaid the loan, the homebuyer's personal obligation to the lender was discharged when the lender released the mortgage.

An investor purchased undeveloped land with the aid of a loan from a bank. The loan, which was evidenced by a note, was secured by a mortgage on the property. The note contained a "due on sale" clause. Subsequently, the investor sold the land to a developer. The bank agreed to waive the "due on sale" clause if the developer assumed the mortgage, which the developer did; the waiver was silent as to the note. The following year, the developer failed to make timely payments on the note, resulting in a default. The developer filed for bankruptcy, and his personal liability for the note was completely discharged. Would the bank be successful in an action against the investor based on the default of note? Answers: A: Yes, because the developer is no longer liable on the note. B: Yes, because the investor is liable on the note. C: No, because the bank waived the "due on sale" clause. D: No, because the developer assumed the mortgage.

Answer choice B is correct. Following the transfer of a mortgage, the investor, as maker of the note, remains liable on the note absent a release by the bank or other act that would discharge the investor's liability. Answer choice A is incorrect because the investor's liability to the bank for the default does not depend on the developer's liability. A discharge of the developer does not discharge the investor. Answer choice C is incorrect because although the bank waived the "due on sale" clause, which would have triggered the investor's liability for the full amount of the loan upon the sale of the land, the bank did not release the investor from liability on the note. Answer choice D is incorrect because assumption of a mortgage by a transferee of the mortgaged property does not eliminate the transferor's liability, but instead makes the transferee also liable to the mortgagee for payment of the note. The elimination of the transferee's personal liability through bankruptcy does not absolve the transferor of liability with respect to the note.

A plaintiff filed a complaint in state court based on a negligence claim arising from an accident. Before the defendant responded to the complaint, the plaintiff voluntarily filed a notice of dismissal. Subsequently, the plaintiff filed a complaint in federal district court based on diversity jurisdiction, asserting the same claim. The defendant filed a motion to dismiss for improper venue, and the court denied this motion. Before the defendant filed an answer to the complaint, the plaintiff again voluntarily filed a notice of dismissal. Can the plaintiff later pursue this claim through an action filed in federal court? Answers: A: No, because the second dismissal occurred after the defendant had filed a motion to dismiss for improper venue. B: No, because the second dismissal was with prejudice. C: Yes, because both dismissals were voluntary. D: Yes, because the first dismissal was a state action, not a federal action.

Answer choice B is correct. If a plaintiff has voluntarily dismissed an action based on a claim, a subsequent voluntary dismissal of an action based on the same claim is a dismissal with prejudice and thus has preclusive effect. This is sometimes referred to as the "two-dismissal" rule. Answer choice A is incorrect. Even though the defendant had responded to the second complaint, the plaintiff could voluntarily dismiss the action without court approval or the defendant's agreement because the defendant had not filed an answer or a summary judgment motion at the time that the plaintiff filed the notice of dismissal. However, even though the dismissal was voluntary and court approval or the defendant's agreement was not required, the second dismissal was with prejudice under the two-dismissal rule. Answer choice C is incorrect. Even though both dismissals were voluntary, the second dismissal is with prejudice under the two-dismissal rule. Answer choice D is incorrect because the two-dismissal rule applies regardless of whether the prior action was a state or federal action

The owner of a rare manuscript and a collector entered into a written agreement for the sale of the manuscript at a price of $7,500, which was to be paid upon delivery of the manuscript on a specific day. When the owner tendered the manuscript on the specified date, the collector refused to accept it or to pay for it. The owner tried in good faith, by commercially reasonable means, to sell the manuscript, but he was unsuccessful. He subsequently brought suit against the collector for specific performance of the contract. For whom should the court rule? Answers: A: The collector, because there has been no breach of contract. B: The collector, because damages are an adequate remedy for the owner. C: The owner, because he failed to sell the manuscript in good faith. D: The owner, because specific performance is an available remedy under the Uniform Commercial Code.

Answer choice B is correct. In general, specific performance is not available as a remedy for breach of a contract when damages are an adequate remedy for the nonbreaching party. Because the seller in a sale-of-goods contract typically receives money from the buyer in exchange for the goods, damages are an adequate remedy for the seller, as they would be in this instance for the manuscript owner. Answer choice A is incorrect. Although the collector did not take the manuscript when the owner tendered it to the collector on the date specified in their contract, the collector's refusal to pay for the manuscript at that time, as specified in the contract, constituted a breach of the contract. Answer choice C is incorrect. Although the owner has been unsuccessful in his attempts to sell the manuscript despite doing so in good faith and in a commercially reasonable manner, this failure does not prevent damages from being an adequate remedy for him. It simply means that his damages as measured by his inability to resell the manuscript would be the contract price of the manuscript. Answer choice D is incorrect. Although the Uniform Commercial Code (UCC) lists specific performance as a remedy for the buyer of goods, the UCC does not list specific performance as a corresponding remedy for the seller of goods. As noted with respect to answer choice B, damages are an adequate remedy for a seller who, pursuant to the contract, is to receive money from the buyer for the goods.

A developer purchased a parcel of oceanfront property to build a resort on it. After the developer purchased the land and began contacting contractors about building the resort, the county enacted a local ordinance requiring that the owners of all oceanfront property take affirmative steps to prevent shore erosion. The ordinance requires that, "to preserve the unique ecosystem and ecological flood-protection benefits of the sand dunes, all owners of oceanfront property must promote the formation and maintenance of natural sand dunes by planting beach grass at a designated distance from the water and by erecting sand fencing to prevent damaging foot traffic." The regulation, if enforced, would force the developer to build a far smaller, albeit still profitable, resort much farther from the ocean than he had planned. The developer sued the county, claiming that the ordinance constitutes a taking of the developer's oceanfront property and that the county therefore owes the developer just compensation. Is the court likely to rule in favor of the developer? Answers: A: No, because a taking can occur only when the government takes title to private property. B: No, because a governmental regulation that adversely affects a person's property interest is generally not a taking. C: Yes, because the conservation objective of the county ordinance is not sufficiently compelling to justify the substantial diminution in the property value. D: Yes, because the county's ordinance will constitute a taking to the extent that the developer's resort will be less profitable.

Answer choice B is correct. In the context of a regulation, a state or local government can act under its police power for the purposes of health, safety, welfare, aesthetic, and environmental concerns. A taking occurs when the government takes title to land, physically invades land, or severely restricts the use of land. Generally, a governmental regulation that adversely affects a person's property interest is not a taking. Here, the developer can still build a smaller resort and make a profit. Consequently, it is unlikely that the court will find that a taking has occurred. Answer choice A is incorrect because the government's acquisition of title to property is not necessary to establish a taking within the meaning of the Fifth Amendment. Answer choice C is incorrect because a court generally does not assess whether an objective is "sufficiently compelling" when determining whether a governmental action is a taking within the meaning of the Fifth Amendment. The government need only have a legitimate state interest. Answer choice D is incorrect because the mere fact that a governmental regulation causes a drastic reduction in the value of the property is not by itself grounds for finding that a taking has occurred, much less a governmental regulation that results in a mere reduction in the profitability of a particular property.

A student at a large state university decided to enter a classroom full of students with a gun to demand that the professor give the student a passing grade in a course. The student entered the packed classroom with a machine gun and hundreds of extra rounds of ammunition. The professor, seeing him from the podium, activated a silent alarm at the podium, and the police arrived within seconds. The student, realizing he needed to escape, fired several shots toward the front of the classroom so that the police would not come in. He then ran out the back door of the classroom into an alley. One of the shots he fired hit a member of the class sitting in the fifth row and killed her. In this jurisdiction, murder in the first degree is defined as "intentionally killing another human being with deliberation." Deliberation is "even one moment of planning or cool reflection." Second-degree murder includes all other kinds of murder except for felony murder. The crime of common-law felony murder is codified by statute as murder in the third degree. The jurisdiction follows the common-law definitions for manslaughter. What is the most serious crime for which the student can be found guilty? Answers: A: Murder in the first degree. B: Murder in the second degree. C: Murder in the third degree. D: Manslaughter.

Answer choice B is correct. In this jurisdiction, all murders that do not fall into the categories of first or third degree are murders in the second degree. The student's actions in firing a gun in the classroom showed a wanton and willful disregard of an unreasonable risk to human life, which amounts to a depraved heart killing. Therefore, the student can be found guilty of second-degree murder. Answer choice A is incorrect because the student cannot be found guilty of murder in the first degree as defined by this statute. When he fired the gun, he did not intend to kill anyone, but rather to create a means of escape. Answer choice C is incorrect. The jurisdiction has codified the common-law rule for felony murder as murder in the third degree. Felony murder is an unintended killing proximately caused by and during the commission or attempted commission of an inherently dangerous felony. Under the common law, the following felonies are considered inherently dangerous: burglary, arson, rape, robbery, and kidnapping. In this case, the student cannot be found guilty of felony murder because he has not committed or attempted to commit any of the inherently dangerous felonies required for felony murder. Answer choice D is incorrect because manslaughter is a less serious crime than murder in the second degree.

A dog sitter customarily went into her clients' homes, with keys they provided to her, to care for their pets. Over the years, she grew to especially love one particular dog. She would regularly enter the home of her client on days she was not scheduled to watch the dog and would take the dog to the park. One afternoon, the dog she loved wandered into her yard as she was doing yard work. She did not immediately call her client, but instead decided to keep the dog for a few days. A week later when she returned him to the client, the client stated that she so enjoyed having the house to herself that she was going to sell him. The dog sitter immediately purchased him and brought him back to her home. Were the dog sitter's actions sufficient to meet the requirements of larceny? Answers: A: No, because she had a key to enter the home. B: No, because she was only going to keep the dog for a few days. C: Yes, because she kept the dog despite knowing who owned it. D: Yes, because she would enter the client's home regularly and take the dog to the park.

Answer choice B is correct. Larceny is the trespassory taking and carrying away of the personal property of another with the intent to permanently deprive that person of the property (i.e., intent to steal). The property must be taken without the owner's consent. Although the dog sitter did retain the property of another with the intent to deprive the owner of the dog temporarily, she never intended to keep him permanently until the client sold him to her. Answer choice A is incorrect because, although she had permission to enter the home and watch the dog, that would not give her the authority to permanently deprive the client of the dog. Answer choice C is incorrect because she never had the intent to permanently deprive her client of the dog. While it is possible for a larceny to be committed even if the intent to steal is not formed until after the taking under the continuing trespass rule, here, the dog sitter did not intend to keep the dog permanently until after she was given permission to do so. Therefore, since the dog's owner consented to the dog sitter keeping the dog, there was no continuing trespass. Answer choice D is incorrect because the act, while problematic, was not larceny. She was certainly trespassing, and could likely be found guilty of some crime, as the key was given only to enter when she was dog-sitting, but she did not intend to permanently deprive the owner of the dog. Accordingly, she could not be found guilty of larceny.

A businesswoman was walking to her car. When she got to the car, a hooligan stepped out from an alley and approached her with his hands in his pockets. He smiled, took a set of keys out of his pocket, and said, "Nice car. It would be a real shame if someone keyed it and knocked off its side mirrors, wouldn't it? The world is just full of maniacs who'll break anything, you know?" Frightened, the woman threw up her hands and dropped her purse. The hooligan grabbed the purse from the ground and ran away. When he turned a corner, the hooligan heard sirens behind him. Afraid that a cop might have observed the incident, he threw the purse in a dumpster and kept running. He was later arrested. The crimes below are listed in descending order of seriousness. What is the most serious crime of which the hooligan can be convicted? Answers: A: Robbery. B: Larceny. C: Attempted robbery. D: Attempted larceny.

Answer choice B is correct. Larceny requires a trespassory taking and carrying away of the personal property of another with the intent to steal it. All the elements of larceny were satisfied here. The "carrying away" (i.e., asportation) element requires movement of only a slight distance. This element was satisfied here even though the hooligan ultimately discarded the purse. Answer choice A is incorrect because robbery is larceny by force or intimidation when the taking of the property is from the person or presence of the victim. Although the purse was still legally within the businesswoman's presence after she dropped it, the taking of property by intimidation must be the result of a threat of immediate serious physical injury to the victim, a close family member, or other person present. A threat to damage or destroy property, other than the victim's home, is probably not sufficient. Answer choice C is incorrect because the hooligan could be convicted of the more serious charge of larceny. In addition, there is no evidence that the hooligan was attempting to make a qualifying threat for a robbery. Answer choice D is incorrect because, although the hooligan can be convicted of attempted larceny, the hooligan can be convicted of the more serious completed crime of larceny.

The driver-owner of an automobile that was involved in an accident with a truck sued the driver of the truck in federal court based on diversity jurisdiction for personal injuries and property damages suffered by the driver-owner as a result of the accident. A judgment was entered based on a jury verdict that the truck driver was not at fault for the accident. Subsequently, the passenger in the automobile at the time of the accident, who was unrelated to the driver-owner of the automobile, sued the truck driver in the same federal court based on diversity for personal injuries suffered by the passenger as a result of the accident. Can the truck driver successfully preclude the re-litigation of his fault in the accident? Answers: A: No, because the judgment in the prior case was based on a jury verdict. B: No, because the passenger was not a party to the prior lawsuit. C: Yes, because defensive use of issue preclusion is permitted. D: Yes, because the issue of the truck driver's fault for the accident was litigated in the prior lawsuit.

Answer choice B is correct. One of the requirements for the use of issue preclusion is that the party against whom it is to be asserted in the current lawsuit was a party (or one in privity with a party) to the prior lawsuit. In this case, because the passenger was not a party or in privity with a party to the prior lawsuit between the driver-owner and the truck driver, the truck driver may not successfully assert that the determination of his fault with regard to the accident in that lawsuit precludes the litigation of that issue in the current lawsuit. Answer choice A is incorrect. An issue determined by a jury can preclude re-litigation of that issue in a subsequent trial. Although issue preclusion may only apply if the issue to be precluded was essential to the judgment, and it is not always possible to determine whether the issue was essential to a jury's verdict, the jury in this prior case clearly found that the truck driver was not at fault for the accident, and the judgment was based on that finding. Answer choice C is incorrect. Although defensive use of issue preclusion is permitted, and the truck driver in the current lawsuit is pleading issue preclusion as an affirmative defense, he cannot successfully assert this defense against the passenger who was not a party to the prior lawsuit. Answer choice D is incorrect. Although one of the requirements for the use of issue preclusion is that the issue was actually litigated in the prior lawsuit, the party against whom it is to be asserted in the current lawsuit must also have been a party to the prior lawsuit. This requirement has not been met on these facts.

A beautiful woman preyed upon lonely, desperate men looking for love on the Internet by getting a potential suitor to fall in love with her and then propose marriage to her. After a suitor would propose to her and give her an engagement ring, the woman would have a replica of the engagement ring made at a fraction of the cost of the real one. Shortly thereafter, the woman would break off the engagement, return the knock-off version of the engagement ring to the duped suitor, and then sell the real engagement ring for a sizeable profit. After scamming her most recent suitor, the suitor tried to return the knock-off ring to a high-end jewelry store the same day that he was dumped. The jewelry store marked all of their rings with a very specific insignia that could not be replicated. Once they realized that the ring was a knock-off, they called the police and the woman was arrested. If the woman is guilty of any crime, she is most likely guilty of: Answers: A: Embezzlement B: False pretenses C: Larceny D: Larceny by trick

Answer choice B is correct. The crime of false pretenses is a false representation of a past or present material fact made by the defendant with the intent to cause the victim to pass title to the defendant, and title is passed. False pretenses requires obtaining title to the property of another person through the reliance of that person on a known false representation of a material past or present fact, and the representation is made with the intent to defraud. Here, the suitor relied on the false representation that the woman loved him and intended to marry him, and the woman made this representation with the intent to defraud the suitor and obtain title to the engagement ring. Answer choice A is incorrect. Embezzlement is the fraudulent conversion of the property of another by a person who is in lawful possession of the property. The woman had possession of the engagement ring because the suitor gave her the ring as a symbol of their agreement to marry. Additionally, the woman had the intent to defraud the suitor and convert the property because she returned a knock-off version of the engagement ring and sold the real one. However, because she obtained title to the ring through a false representation—that she loved the suitor and intended to marry him, the woman will more likely be convicted of false pretenses. Answer choice C is incorrect. Larceny is the trespassory taking and carrying away of the personal property of another (without his consent) with the specific intent to permanently deprive the owner of the property at the time of the taking. Here, there was no trespassory taking and caring away of the engagement ring from the suitor without his consent, so there was no larceny. Answer choice D is incorrect. Larceny by trick occurs when the defendant fraudulently induces the victim to deliver possession of the property to the defendant. Here, the woman obtained title to the engagement ring, not just possession, so she is most likely guilty of false pretenses.

Two homeowners who own adjacent lots decided to construct a driveway that straddles their common property line. They entered into a written agreement whereby each granted to the other a right of access over his property in accord with the dimensions and location of the driveway and each promised that he, his heirs, devisees, assignees and successors would equally share the repair and maintenance of the driveway, including the cost of snow removal, with the neighboring property owner. This agreement was promptly and properly recorded. After the driveway was built, the owner of one of the improved lots sold it in fee simple absolute to a buyer who currently lives on the premises. A year ago, the owner of the other lot died. His property passed by intestacy to his son. Through an oral agreement, the son, who has a residence elsewhere, has rented out the inherited residence to a tenant for a one-year term. The son told the tenant of the shared maintenance agreement with the neighbor, but otherwise did not discuss the maintenance of the driveway with the tenant. This past winter, due to a number of heavy snowfalls, the removal of the snow in order to be able to use the driveway was costly. When the tenant contacted the current owner of the neighboring property, the owner, noting that he rarely used the driveway due to his advanced age and poor health, refused to reimburse the tenant for the snow removal costs borne by the tenant. If the tenant seeks to recover half of the costs for snow removal from the current neighbor, who will prevail? Answers: A: The tenant, because the tenant acted in reliance to his detriment on the son's revelation of the agreement. B: The tenant, because the agreement contains a covenant that runs with land. C:The neighbor, because only the son can enforce the agreement. D: The neighbor, because there was no horizontal privity between the neighbors who entered into the agreement.

Answer choice B is correct. The elements for both the burden to run (i.e., writing, intent, horizontal and vertical privity, notice, and "touch and concern") and the benefit to run (i.e., writing, intent, limited vertical privity, and "touch and concern") are present. Answer choice A is incorrect because the doctrine of detrimental reliance is not applicable. The tenant did not rely on a promise made to him by the current neighbor. Answer choice C is incorrect because, although strict vertical privity is required for the burden to run, which exists in this case because the current owner took the property in fee simple absolute from the promisor, the benefit may run to the current possessor who enjoys a lesser estate than the promisee. The fact that the tenant has only a tenancy for years (in this case, one year) will not prevent the tenant from being able to enforce the promise. Answer choice D is incorrect because, although generally an agreement between neighbors cannot be a covenant that runs with the land because they do not share a common estate, the neighbors by granting each other an easement as part of the agreement, have a shared common estate with regard to the property subject to the easements.

At a jury trial for attempted murder, the prosecutor sought to introduce the testimony of the detective who had interrogated the defendant following his arrest. After many long and grueling hours of questioning, the detective obtained a confession from the defendant. The defendant contended that his confession was involuntary. Accordingly, the defendant sought a hearing outside the presence of the jury to determine the admissibility of the confession prior to the detective's testimony. Must the court grant the defendant's request? Answers: A Yes, because the request involves a preliminary question of the competency of evidence. B: Yes, because the request involves the admissibility of a confession. C: No, because the decision to grant a hearing is within the court's discretion. D: No, because the admissibility of confessions is determined by the jury.

Answer choice B is correct. The trial judge generally decides preliminary questions regarding the competency of evidence, including the admissibility of evidence, whether a privilege exists, and whether a person is qualified to be a witness. Hearings on preliminary matters must be conducted outside the presence of the jury when the hearing involves the admissibility of confessions, when a defendant in a criminal case is a witness and so requests, or when justice requires it. In this case, the defendant sought a hearing outside of the presence of the jury regarding the admissibility of his confession. Accordingly, the court must grant the defendant's request and the judge will decide whether the confession is admissible. Answer choice A is incorrect. Hearings on preliminary matters must be conducted outside the presence of the jury only when the hearing involves the admissibility of confessions, when a defendant in a criminal case is a witness and so requests, or when justice requires it. Answer choice C is incorrect because the court must grant a defendant's request for a hearing outside the presence of the jury when it involves the admissibility of a confession. Answer choice D is incorrect because the trial judge generally decides the admissibility of evidence, such as a confession.

The Department of Defense entered into a contract with a state-owned weapons manufacturer. The contract called for the production of defense equipment to be used by the military. The Department of Defense agreed to purchase all of the defense equipment produced by the state-owned weapons manufacturer for a period of one year. Each purchase made by the Department of Defense during that one-year period was subject to taxation pursuant to state law. The Department of Defense filed suit in federal court challenging the constitutionality of the state tax applied to its purchases under the contract. Is the Department of Defense subject to the state tax for its purchase? Answers: A: No, because the contract involved military defense. B: No, because the Department of Defense is immune from state taxation. C: Yes, because the power to tax is an essential incident of state sovereignty. D: Yes, because, by purchasing defense equipment from a state-owned manufacturer, the Department of Defense consented to state taxation.

Answer choice B is correct. The federal government and its instrumentalities (such as a national bank) are immune from taxation by the states. In this case, the Department of Defense, part of the federal government, is immune from taxation by the state for any purchases made pursuant to the defense contract. Answer choice A is incorrect. The mere fact that the contract involved military defense is not enough to result in immunity from state taxation. The key fact here is that the Department of Defense is the purchaser under the contract, and thus it is immune from the state tax. Answer choice C is incorrect. Although the power to tax is an essential incident of state sovereignty, that power is limited when applied to the federal government. Answer choice D is incorrect. The Department of Defense did not consent to state taxation merely by entering into a contract for defense equipment from a state-owned manufacturer. It is immune from state taxation regardless of whether the other party to the contract was the state or a private entity.

A driver was traveling in his car one evening when he heard his cell phone ring. The phone was located on the console of the car, and the driver reached out to grab it. While the driver was looking at his caller identification display, a pedestrian walked into the crosswalk straight ahead of the vehicle. When the driver saw the pedestrian in the crosswalk, he swerved to the right to avoid the collision; in doing so, he slammed his car into a utility pole. The utility pole crashed to the ground. A power line attached to the utility pole snapped in two, resulting in power outage within the surrounding area. One block away from the accident site, a child was watching television when the power went out, and her house became pitch black. The child, who was scared of the dark, ran out of the house and into the street where the child was struck by a bicyclist and seriously injured. The child's parents sued the driver of the car that crashed into the utility pole, claiming that the child's injury was attributable to the driver's negligence. Who should prevail? Answers: A: The driver, because his actions were not the cause in fact of the child's injuries. B: The driver, because he owed no duty to the child. C: The parents, because the driver's conduct was the legal and proximate cause of the child's injuries. D: The parents, because the driver owed a duty of reasonable care to the child.

Answer choice B is correct. The majority of jurisdictions follow the "Cardozo view" of duty. Under this view, a defendant is liable only to a plaintiff who is within the "zone of foreseeable harm." Here, the child was one block from the accident site, and her injuries occurred in a way that was unforeseeable at the time of the accident. Thus, she is an "unforeseeable plaintiff," and the driver owed her no duty of reasonable care. Answer choice A is incorrect because but for the driver's actions, the child would not have been injured. Answer choice C is incorrect because even if it were true (which it very likely is not), it ignores the issue of duty, which must be addressed before causation. Answer choice D is incorrect because it articulates the "Andrews view," which is followed only by a minority of jurisdictions. That view posits that, if if the defendant can foresee harm to anyone as a result of his negligence, then a duty is owed to everyone (foreseeable or not) harmed as a result of his breach.

A woman sent an offer to sell her office printer to her friend for $450. In her offer, the woman mentioned that an acceptance could be mailed to her business address, and that the friend should let her know within the next couple of weeks whether she was interested. The friend needed an office printer, so she immediately accepted the woman's offer by mailing a letter to the woman's home address. The woman only checked her mailbox at home once a week because she received so much junk mail, so she did not see the acceptance letter. Thinking that her friend was not interested, the woman sold her office printer to a different person. The next day, the friend came to the woman's house with a check for $450. The woman told the friend she had already sold the office printer. Will the friend succeed in an action for breach of contract? Answers: A: Yes, because the offer was irrevocable for at least two weeks. B: Yes, because the woman did not specify that mailing an acceptance to her business address was the only mode of acceptance. C: No, because it was not reasonable for the friend to mail her acceptance to the woman at the woman's home address. D: No, because the woman did not see the friend's acceptance letter.

Answer choice B is correct. The offeror can dictate the manner and means by which an offer may be accepted. Unless the offeror specifically requires the offeree to accept in a particular manner or by using a particular means, the offeree can accept in any reasonable manner and by any reasonable means. Here, the woman did not specify that the acceptance could only be mailed to her business address; she merely suggested that an acceptance could be mailed to her business address. Thus, another reasonable mode of acceptance, such as mailing it to the woman's home address, is permissible. Accordingly, answer choice C is incorrect. Answer choice A is incorrect because offerors may generally revoke offers at any time prior to acceptance, even if the offeror claims that the offer will be held open. Here, the friend would not succeed because the offer was irrevocable, but because she sent her acceptance before the offer was revoked. Answer choice D is incorrect. Under the mailbox rule, an acceptance is valid upon posting, not receipt. Here, the friend's acceptance was effective when she mailed the letter to the woman's home. Upon mailing the letter, an enforceable contract was created between the woman and the friend, even though the woman did not see the friend's acceptance letter.

A buyer contracted with an owner of commercial property located in a strip mall to purchase the property for $750,000. The contract called for closing to occur on March 1. Prior to that date, the owner informed the buyer that the current tenants were wrongfully refusing to vacate the premises, and would not do so until March 31. The owner notified the current tenants, who ran a call center on the premises, that they would need to vacate the premises before April 1. Although the current tenants stopped operating the call center before April 1, they were not able to empty the space completely because they had attached numerous cubicles to the floor, and the cubicles occupied the entire space of the property. The buyer, who unbeknownst to the owner, planned on using the property as a gymnastics studio, had ordered gymnastic equipment that was to be delivered on March 2. Due to the delay, the buyer was forced to rent a storage unit for this equipment for $1,000. By April 1, the fair market value of the property had risen to $755,000. In addition, the monthly fair market rental value of the property was $3,000. If the buyer files an action against the owner for damages, what will she likely recover? Answers: A: $1,000 in storage unit costs, and $3,000 for the fair rental value of the property for the month of March. B: $3,000 for the fair rental value of the property for the month of March. C: Nothing, because the property increased in value during the month of March. D: Nothing, because the delay was attributable to the tenants, not the owner.

Answer choice B is correct. The owner breached the contract by failing to close on the property on March 1. As a result, the owner is liable to the buyer for damages that the buyer suffered because of the breach. The buyer was denied possession of the property for the month of March. Therefore, the buyer is entitled to the fair market rental value of the property for a month, which is $3,000. Answer choice A is incorrect. The owner was unaware of the buyer's intended use of the property. Therefore, the buyer is not entitled to consequential damages for the cost of storing the gymnastic equipment for the month. Answer choice C is incorrect because the buyer would have paid the same price, and received the same benefit, regardless of the owner's breach, which makes the increased value of the property insufficient compensation for the loss the buyer suffered. Answer choice D is incorrect. Although the owner may seek damages from the tenants for wrongfully holding over, the owner is nevertheless liable to the buyer for his own failure to deliver the property to the buyer on March 1, as contractually promised.

A defendant is on trial for armed robbery and felony murder. The prosecution seeks to admit testimony by the first witness to arrive at the scene of the crime. The witness discovered the victim just before he died of a gunshot wound, and the victim identified the defendant as his assailant only moments before he died. The defense asserts that the victim was too delirious from blood loss to know that he was dying, and hopes to present a statement from the victim's widow to support this assertion. However, both parties agree that the widow's statement is privileged under federal law. In what manner should the court determine whether the victim's statement is a dying declaration? Answers: A: Allow the prosecution to admit the testimony only if the judge determines that the witness is credible. B: Consider only the unprivileged evidence from both sides outside the presence of the jury. C: Consider all of the evidence from both sides outside the presence of the jury. D: Allow only the unprivileged evidence from both sides at the jury trial so the jury may decide whether the victim believed he was dying.

Answer choice B is correct. The trial judge generally decides preliminary questions regarding the competency of evidence, including the admissibility of evidence, whether privilege exists, and whether a person is qualified to be a witness. Hearings on preliminary matters must be conducted outside the presence of the jury when the hearing involves the admissibility of confessions, when a defendant in a criminal case is a witness and so requests, or when justice requires it. Here, the determination of the admissibility of a statement by the victim identifying the defendant as his assailant is clearly a matter that justice requires that the jury not be present. The court is not bound by the Federal Rules in deciding these questions, except with respect to privileges. Accordingly, even the judge cannot consider the privileged evidence. Answer choice A is incorrect because determinations of credibility must be made by a jury. Answer choice C is incorrect because the court is bound by the Federal Rules in deciding preliminary questions of admissibility with respect to privileges. Answer choice D is incorrect because this preliminary question must be decided by the judge, not the jury.

A defendant was charged with two separate crimes related to vandalism in a national park. Each of the crimes was punishable by a maximum of six months imprisonment and a $5,000 fine. The defendant requested a jury trial, but his request was denied. The defendant proceeded to trial before a judge in federal court, and was subsequently convicted on both charges. The judge sentenced the defendant to five months of imprisonment for each charge, to be served consecutively, as well as a fine of $5,000 for each charge. The defendant appealed his conviction, arguing that he was entitled to a jury trial. Was the defendant entitled to a jury trial? Answers: A: No, because the defendant's actual sentence for each offense was less than six months. B: No, because the maximum sentence for each offense was six months. C: Yes, because the combined maximum sentence for the offenses was 12 months. D: Yes, because the sizeable fine makes each crime a serious offense.

Answer choice B is correct. There is a right to a jury trial for offenses that carry an authorized sentence of more than six months, regardless of the actual penalty imposed. In this case, the maximum sentence for each offense was only six months, and thus the right to a jury trial was not triggered. Answer choice A is incorrect because the actual sentence imposed is irrelevant in determining whether the right to a trial by jury attaches; rather, the right hinges on the maximum authorized sentence. Answer choice C is incorrect because the maximum prison term for each offense cannot be aggregated. Thus, there is no right to a trial by jury for multiple petty offenses that carry a combined total maximum term exceeding six months. Answer choice D is incorrect because a maximum fine of $5,000 in addition to six months imprisonment does not convert the crime into a serious offense.

Concerned with the proliferation of signs about upcoming events and the failure to remove those signs after the event, a city enacted an ordinance that limited the number of such signs that could be displayed on public property and set a time period before and after an event during which signs about that event could be displayed. A social organization wants to display signs about its monthly dinner, which is held to attract new members, in greater number and for a longer period than permitted by the ordinance. The organization has filed a lawsuit, challenging the constitutionality of the ordinance. Of the following, by which standard will this ordinance be judged? Answers: A: It must be narrowly tailored to further a significant government interest, and leave open alternative channels of communication. B: It must be the least restrictive means for accomplishing a compelling governmental interest. C: It must be rationally related to a legitimate government interest. D: It must not have a negative impact on the organization's freedom of assembly.

Answer choice B is correct. This regulation is a restriction on speech that is content-based on its face. The regulation applies to one specific topic (upcoming events) and subjects that speech to different--and more onerous--requirements than any other signs. Therefore, the city's ordinance will be judged by a strict scrutiny standard. Answer choice A is incorrect because it states the standard for time, place, and manner restrictions. However, this is a content-based restriction, so strict scrutiny applies. Answer choice C is incorrect because it states the rational basis test, which does not apply to content-based restrictions on speech. Answer choice D is incorrect. All constitutional protections, including the First Amendment protection of the freedom of assembly, are not absolute. The mere fact that governmental conduct has a negative impact on this right is not sufficient to preclude the constitutionality of an ordinance.

On May 1, a clothing manufacturer sent a written offer to a retailer for the sale of 1,000 pairs of designer jeans at a price of $50 per pair, including delivery costs. The proposed delivery date was June 15. On May 5, the retailer mailed a letter accepting the manufacturer's offer. On May 15, a natural disaster occurred causing fuel prices to significantly increase. As a result, the clothing manufacturer sent a letter to the retailer requesting an additional $500 to cover the increased delivery and transportation costs for the shipment of jeans. The retailer considered seeking out a new supplier, but hoped to continue doing business with this manufacturer in the future, so decided against it. The retailer returned a signed letter promising to pay the additional $500. On June 15, the clothing manufacturer delivered 1,000 pairs of jeans to the retailer. The retailer paid the clothing manufacturer $50,000, but refused to pay the additional $500. Can the manufacturer enforce the retailer's promise to pay an additional $500? Answers: A: Yes, because the manufacturer relied on the promise when it delivered the jeans to the retailer. B: Yes, because the manufacturer made its request for the additional $500 in good faith. C: No, because the attempted modification was unconscionable. D: No, because no consideration supported the promise to pay the additional $500.

Answer choice B is correct. Unlike under the common law, under Article 2, no consideration is necessary to modify a contract; however, good faith is required. Good faith requires honesty in fact and fair dealing in accordance with reasonable commercial standards. If a party demands an increase in price because the other party has no choice but to agree, the courts will invalidate such a bad-faith modification. Here, a natural disaster occurred causing fuel prices to significantly increase. As a result, the clothing manufacturer's request for an additional $500 to cover the increased delivery and transportation costs was made in good faith, based on the natural disaster. Also, the retailer was not forced into the agreement because other suppliers were available. Therefore, the attempted modification was valid, and the manufacturer can enforce the retailer's promise to pay an additional $500 without reliance or additional consideration. Answer choice A is incorrect. Promissory estoppel is not necessary here because there was an enforceable modification of the contract. Thus, the manufacturer can enforce the promise under the valid contract and need not show reliance. Answer choice C is incorrect because the request for an additional $500 does not make the modified contract unconscionable. A contract (or part of a contract) is only unconscionable when it is so unfair to one party that no reasonable person in the position of the parties would have agreed to it. Answer choice D is incorrect because contracts under the UCC, such as the one here, do not require consideration for modification; rather, only good faith is required.

The owner of a rare eighteenth-century chest offered to sell it to a connoisseur of antiques for $75,000. The connoisseur countered that she would buy the chest for $50,000. The owner rejected this price. The owner and the connoisseur then executed a written agreement for the sale of the chest at a price to be decided by a local antiques dealer whom they both trusted. The dealer examined the chest. He told the owner and the connoisseur that he had to do further research on the chest, but that he would let them know his decision in several days. Unfortunately, the dealer died before doing so. A reasonable price can be established for the chest by the court. Is there likely an enforceable contract? Answers: A: No, because the price of the chest was not determined at the time the agreement was executed. B: No, because the owner and the connoisseur did not intend to be bound unless the dealer set the price of the chest. C: Yes, because a reasonable price can be established for the chest by the court. D: Yes, because the owner and the connoisseur executed a written agreement for the sale of the chest.

Answer choice B is correct. When parties enter into an agreement for the sale of goods and the price has not been set, there is no contract if the agreement reflects an intent not to be bound unless the price is subsequently set and the price is never set. Here, the owner and the connoisseur agreed to the sale of the chest at a price set by the dealer. Because the dealer did not set a price before his death, there is no enforceable agreement for the sale of the chest. Answer choice A is incorrect because such an agreement may be enforceable despite an unresolved price at the time that the agreement is entered into, as long as the price is arrived at in the manner agreed to by the parties. Here, had the dealer named a price for the chest, there would have been a contract that could have been enforced by either party against the other. Answer choice C is incorrect. Although a court may fix a reasonable price for goods when the parties to a contract fail to do so, the parties must intend to enter into a contract for the goods despite the absence of the price term. Here, the owner and the connoisseur did not ignore the price term, but rather set out a specific method by which the price was to be determined. Because, due to the death of the dealer, that method cannot be used to determine the price, and the parties did not provide for an alternative method, the implication is that the parties did not intend to be bound if that specified method failed. Answer choice D is incorrect. Although the owner and the connoisseur did execute a written agreement, this agreement left the price of the chest to be determined by the dealer. Because the dealer's death makes it impossible for him to do so, the agreement fails. The parties' disagreement over the price of the chest prior to entering into the agreement also suggests that the parties did not have the intent to be bound if the dealer were unable to determine a price.

A group of plaintiffs who claimed to have been injured by a corporate defendant's negligent design of its product sought certification as a class in federal district court for a class-action lawsuit against the corporate defendant. The federal district court entered an order certifying the class on August 1. The corporate defendant seeks to appeal the certification of the class. Which of the following is most accurate regarding an appeal by the corporate defendant? Answers: A: The defendant is precluded from filing an appeal of the order because of the final judgment rule. B: The defendant is precluded from filing an appeal of the order because the order is interlocutory. C: If the defendant timely files a petition for permission to appeal, the court of appeals may hear the appeal in its discretion. D: The defendant may appeal the order as of right.

Answer choice C is correct. A court of appeals may permit an appeal from an order granting or denying class action certification if a petition for permission to appeal is filed with the circuit clerk within 14 days after the order is entered. The court of appeals has discretion to deny the appeal. Answer choice A is incorrect. While the final judgment rule would normally prohibit the appeal of an interlocutory order, Rule 23(f) allows for the discretion to permit an appeal regarding class action certification. Answer choice B is incorrect for the same reason. Answer choice D is incorrect. There is no right of appeal here. Rather, the court of appeals may permit an appeal if a petition for permission to appeal has been filed with the circuit clerk within 14 days after the order is entered.

A musician's brother borrowed the musician's electric guitar. The brother told the musician that he was going to use it the next day to teach the children in his second grade class about different musical instruments. Instead, the brother went to a rowdy bar that evening and played the electric guitar with his rock band all night long. A drunk bar patron got on stage during one of the songs, grabbed the guitar from the brother, and smashed it against the ground. The brother took the electric guitar to a repair shop the next day, and was told that it would cost $750 to restore the guitar to its original condition. At the time of the incident, the guitar was worth $1,500. The musician asserts a claim against his brother for conversion. What is the musician entitled to recover? Answers: A: Nothing B: $750 C: $1,500 D: $1,500 plus damages for loss of use

Answer choice C is correct. A defendant is liable for conversion if he intentionally commits an act depriving the plaintiff of possession of her chattel or interfering with the plaintiff's chattel in a manner so serious as to deprive the plaintiff of the use of the chattel. The plaintiff's damages are the chattel's full value at the time of the conversion. Accidentally damaging the plaintiff's chattel is not conversion if the defendant had permission to be using the property. In this case, although the brother had permission to use the guitar, he clearly exceeded the scope of that permission when he took it to a rowdy bar, played it all night long with his rock band, and caused it to sustain significant damage, thus depriving the musician of possession of the chattel in a serious manner. As a result, the musician can recover the full value of the chattel at the time of the conversion ($1,500). Answer choice A is incorrect. The musician can recover the full value of the chattel at the time of the conversion because the elements of conversion have been met. Answer choice B is incorrect because this would be the amount of recoverable damages for trespass to chattels, a separate claim that the musician has not brought. Answer choice D is incorrect because conversion does not entitle the musician to consequential damages.

A personal representative of a decedent's estate was sued by the two beneficiaries of the estate in federal district court for fraud under state law in connection with her handling of the estate. The beneficiaries, who uncovered the alleged fraud after the state probate proceedings had been concluded, each sought damages of $50,000. They were both citizens of the same state as the decedent had been before his death. The personal representative was a citizen of another state. The personal representative has filed a motion to remand the case to state court. Should the federal court grant this motion? Answers: A: Yes, because there is a lack of diversity between the parties. B: Yes, because of the probate exception. C: No, because the case originated in the federal court. D: No, because the amount-in-controversy requirement has been satisfied.

Answer choice C is correct. A federal court can only remand a case to the state court from which the claim originated. Because this action was initially filed in federal court, there is no state court to remand the case to; the federal court lacks the power to order a state court to accept this case. The federal court could, of course, dismiss the case if the court determines that it lacks subject-matter jurisdiction. Answer choice A is incorrect. Whether the action satisfies the "diversity of citizenship" requirement for bringing a state-law based action in federal court turns on whether the personal representative is being sued personally or as a representative of the decedent's estate. If the former, then there is diversity since she lives in a different state from the plaintiffs-beneficiaries. If the latter, then the diversity requirement is not met since she is treated as a citizen of the same state as the decedent. In either case, as noted with respect to answer choice C, remand of the case to state court is inappropriate. Answer choice B is incorrect. The probate exception prohibits a federal district court from interfering with state proceedings to probate or annul a will or to administer a decedent's estate, assuming general control over probate, or affecting possession of property in the custody of a state court. Since the state probate proceedings have concluded, it is likely that this exception would not prevent the federal court from hearing this action. In any case, as noted with respect to answer choice C, remand of the case to state court is inappropriate. Answer choice D is incorrect because, in determining the amount in controversy, the damages sought by each plaintiff generally cannot be aggregated to meet the amount-in-controversy requirement of an amount in excess of $75,000. Since the amount of damages does not cross this threshold, the amount-in-controversy requirement has not been met.

At a trial for burglary, the prosecutor called a witness who lived next door to the victim. The witness testified that on the night of alleged burglary, she heard the victim shout, "[The defendant] has a gun pointed right at me and said he is going to kill me! I'm going to die! Please help!" The victim was not available to testify at trial because the traumatic incident had caused her to suffer from severe anxiety and depression. The defendant objected to the neighbor's testimony on the grounds that it was hearsay. In response, the prosecutor contended the testimony was admissible as a dying declaration. Is the neighbor's testimony admissible as a dying declaration? Answers: A: Yes, because the victim believed that death was imminent. B: Yes, because the victim is unavailable to testify. C: No, because the defendant is being tried for burglary. D: No, because the victim did not actually die.

Answer choice C is correct. A statement qualifies as a "dying declaration" if (i) the declarant believes that her death is imminent, and (ii) the statement pertains to the cause or circumstances of the death she believes to be imminent. Although the declarant must be unavailable, the declarant need not have actually died in order for the statement to avoid exclusion as hearsay. However, the dying-declaration exception applies only in homicide prosecutions and civil cases. Here, the prosecution was for burglary. Therefore, the dying-declaration exception does not apply. Answer choice A is incorrect. Although the victim's statement was made under the belief that her death was imminent and the statement pertained to the circumstances of the death she believed imminent, the dying-declaration exception does not apply to burglary cases. Answer choice B is incorrect. While the victim must be unavailable for the dying-declaration exception to apply, this exception is only available in homicide or civil cases. Answer choice D is incorrect. The declarant need not have actually died in order for the dying-declaration exception to apply.

At the beginning of an auction, which consists of 25 lots of goods owned by the same person, the auctioneer announced that the right of the seller to withdraw goods was not reserved. After the sales of two lots were completed but prior to the auctioneer's call for bids on the third lot, the owner sought to withdraw the third lot from the auction. Can the owner properly withdraw the third lot? Answers: A: No, because the auctioneer had announced that the auction was without reserve. B: No, because sales of other lots owned by the same person had been completed. C: Yes, because the auctioneer has not called for bids on the third lot. D: Yes, because the auctioneer has not announced completion of the sale of the third lot.

Answer choice C is correct. Answer choice C is correct. In an auction, each lot is considered to be a separate sale. In a no-reserve auction, after the auctioneer calls for bids on the goods in a lot, the goods cannot be withdrawn unless no bid is received within a reasonable time. With regard to the third lot, because the auctioneer had not called for bids on that lot, the goods can be withdrawn. Answer choice A is incorrect. Even though the right of the seller to withdraw goods is not reserved, a seller possesses the right to withdraw the goods until the auctioneer has called for bids on the goods. Answer choice B is incorrect. When goods are auctioned in lots, each lot is a separate sale. Consequently, the fact that other lots owned by the same person have been sold in the same auction does not affect the right of the owner of a lot to withdraw that lot from the auction. Answer choice D is incorrect because it states the rule for withdrawal of goods from an auction where the auction is with reserve.

An honest dispute develops between a condominium owner and a plumber over whether plumbing installed in the kitchen and bathrooms of the condominium satisfied contractual specifications. If the plumbing meets those specifications, the condominium owner owes the plumber $15,000 under the terms of the contract. The condominium owner offers to pay the plumber $10,000 in satisfaction of the owner's contractual obligations, if the plumber replaces the plumbing in the kitchen with another grade of pipe. The plumber accepts the condominium owner's offer. After the plumber replaces the kitchen plumbing, the condominium owner refuses to pay the plumber. In a breach of contract action brought by the plumber, the fact-finder determines that the plumbing originally installed by the plumber did satisfy the contract specifications. The fact-finder also determines that the plumber and the condominium owner entered into a substitute agreement for which the owner failed to deliver the required performance. What is maximum amount that the plumber can seek in damages from the condominium owner? Answers: A: $25,000 B: $15,000 C: $10,000 D: Nothing

Answer choice C is correct. Because the plumber and the condominium owner entered into a substitute agreement rather than an accord, the substitute agreement completely replaces the original contract. Consequently, the plumber may only seek damages of $10,000, the amount that the condominium owner promised to pay under that agreement. Had the second agreement between condominium owner and the plumber been an accord, the plumber could have sought damages upon the condominium owner's breach of the accord under either the original contract (i.e., $15,000) or the accord (i.e., $10,000), but not both. For this reason, answer choices A, B, and D are incorrect. NOTE: Be careful with this question. Even though the facts tend to read like an accord, we know that this is instead substitute agreement because the question states that the fact-finder determined this is so.

In a pre-trial hearing, a judge determined that a defendant's confession was given voluntarily to a police detective after the detective had given Miranda warnings to the defendant. At this hearing, the defendant testified. At trial, the defense did not contest the defendant's receipt of Miranda warnings, but sought to question the police detective about the manner in which the defendant was interrogated after receiving the warnings in order to call into question whether the confession was voluntary. The defense does not plan to call the defendant to the witness stand. Should the court permit this line of questioning? Answers: A: No, because there had been a judicial determination that the confession was voluntary. B: No, because the defense did not challenge the defendant's receipt of Miranda warnings. C: Yes, because a party may introduce evidence that is relevant to the weight and credibility of other evidence. D: Yes, because a defendant may testify at a hearing regarding a preliminary question without being required to testify at trial.

Answer choice C is correct. Even though a judge has decided that evidence, such as a confession, is admissible, a party may nevertheless introduce other evidence that is relevant to the weight and credibility of the admitted evidence. Here, the defense seeks to discredit the confession by introducing evidence that it was not given voluntarily. Answer choice A is incorrect because, as noted, a judicial determination as to admissibility of evidence does not foreclose a party from presenting other evidence that challenges the credibility of that evidence. Answer choice B is incorrect because, even though the defense did not contest the defendant's receipt of Miranda warnings, a defendant may challenge a confession given after those warnings as being involuntary. Answer choice D is incorrect because, although a defendant may testify at a hearing on a preliminary question, such as the admissibility of a confession, without being required to testify at trial, this principle is irrelevant to the issue of whether a defendant may present evidence at trial that questions the credibility of admitted evidence.

Two officers, a veteran and a novice, brought an experienced drug-sniffing dog on patrol one evening. They properly pulled over a driver on a bridge for erratic driving, and the veteran waited in the car to let the novice handle the stop. As the novice officer questioned the driver through the driver's window, the dog circled the car and began barking and pawing desperately at the passenger-side door. The novice officer noticed that the driver had a package sitting on his passenger seat. When asked about the package, the driver responded that it was just meat from a butcher shop. As the novice officer stepped away from the window to write a ticket, the driver grabbed the package and threw it out the passenger window and over the railing of the bridge. The dog immediately stopped barking. The veteran officer saw the whole event. Assuming that the parties stipulate that the veteran is an expert in the training and reactions of drug-sniffing dogs, is the veteran officer's testimony describing the dog's reactions to the package admissible in a trial of the driver for possession of illegal drugs? Answers: A: No, because the dog's behavior was assertive conduct, making it hearsay not within any exception. B: No, because the dog's presence at the stop resulted in an improper search. C: Yes, as evidence that the package contained illegal drugs. D: Yes, because evidence generated by a machine or animal falls under an exception to the hearsay rule.

Answer choice C is correct. Evidence generated by a machine or an animal is not hearsay, and is admissible without qualifying under a hearsay exception. Answer choice A is incorrect because the declarant of hearsay must be a person, and therefore the dog's reactions are not hearsay. Answer choice B is incorrect because use of a trained dog to sniff for the presence of drugs is only a search if it involves a physical intrusion onto constitutionally protected property, and here the dog did not intrude on the driver's protected property. Answer choice D is incorrect because, in order to be hearsay, the declarant must be a person. Evidence generated by a machine or an animal is not hearsay, and is therefore admissible whether or not it qualifies under a hearsay exception.

An employee brought an action under Title VII of the Civil Rights Act of 1964 against her corporate employer in federal court for racial discrimination. The employee, from State A, alleged that the corporate employer had denied her a promotion, for which she was clearly qualified, solely on the basis of her race. The corporate employer was incorporated in State B and had its headquarters there, but its main physical presence was in State A. The employee sought $60,000 in damages. The employee sought to join an additional claim for breach of contract against the corporate employer based on a separate business transaction entered into between the employee and the corporate employer. The employee alleged $50,000 in damages arising from the breach. Can the court hear the employee's breach-of-contract claim? Answers: A: Yes, because the claim falls within the supplemental jurisdiction of the court. B: Yes, because the employee may aggregate all claims against the corporate employer to meet the amount-in-controversy requirement. C: No, because the claim does not satisfy the amount-in-controversy requirement. D: No, because the parties are not diverse.

Answer choice C is correct. If the original claim is based on federal-question jurisdiction, then a nonfederal claim may be joined only if diversity jurisdiction exists or if the two claims are part of the same case or controversy as the federal claim such that supplemental jurisdiction applies. In this case, the employee's original claim is based on federal-question jurisdiction because it arose under Title VII of the Civil Rights Act of 1964. Accordingly, supplemental or diversity jurisdiction must exist for the court to be able to hear the employee's additional claim. Supplemental jurisdiction does not apply here because the additional claim is based on a breach of contract relating to a separate business transaction, whereas the original claim is based on racial discrimination relating to a promotion. Thus, the two claims are not part of the same case or controversy. Diversity jurisdiction does not exist in this case. Although the employee and the corporate employer are citizens of different states, the amount-in-controversy requirement is not met. Therefore, the federal court cannot hear the breach-of-contract claim. Answer choice A is incorrect because the two claims do not arise out of the same transaction or occurrence. Answer choice B is incorrect because the employee may only aggregate the claims against the employer to satisfy the amount-in-controversy requirement if the subject-matter jurisdiction over the original claim is based on diversity jurisdiction. Because the original claim here is based on federal-question jurisdiction, the damages sought for each claim may not be aggregated. Answer choice D is incorrect because the employee is a citizen of State A and the corporate employer is a citizen of State B. Although the corporate employer has its main physical presence in State A, it is the location of the corporation's nerve center (i.e., headquarters) that is determinative for diversity purposes.

In the parking lot of a mall, two drivers, while simultaneously backing up, each carelessly struck the other's car. Neither suffered physical injuries, but the damages to the plaintiff's car totaled $10,000; damages to the defendant's car were $1,000. The plaintiff sued to recover the damages to his car and the defendant counterclaimed for the damages to her car. At trial, it was determined that the plaintiff was 70% at fault and the defendant 30% at fault for the accident. The applicable jurisdiction has enacted a modified (i.e., partial) comparative negligence statute. After taking into account her own liability to the plaintiff, how much may the defendant recover from the plaintiff? Answers: A: Nothing B: $300 C: $700 D: $1,000

Answer choice C is correct. In a modified (i.e., partial) comparative negligence jurisdiction, when there is only one defendant, a plaintiff may recover from the defendant to the extent of the defendant's fault, but the plaintiff is precluded from recovering if his fault exceeds the defendant's fault. Because the plaintiff here was 70% at fault, he is liable for 70% of the defendant's damages (i.e., $700). And, because the plaintiff was more at fault than the defendant, the plaintiff cannot recover anything from the defendant for damages to his own car. Consequently, the defendant is entitled to recover $700 from the plaintiff, making answer choices A, B, and D incorrect.

A construction company was removing air conditioning equipment from a truck parked on a residential street, using a crane. The equipment weighed several hundred pounds. A half-block away, the mother of a seven-year-old boy was standing on her front porch, watching her son walk to the school bus stop at the end of the block. As the boy walked past the construction site, the air conditioning equipment fell, landing mere inches away from the boy, who calmly continued walking to his bus. While the boy suffered no ill effects from the incident, the boy's mother, who had a clear view of the incident, immediately fainted after seeing her son so close to certain death. For the next 48 hours, the mother was unable to function due to shock over the event. In the following months, the mother had difficulty sleeping due to nightmares as a result of the incident. The mother sued the construction company on behalf of her son for negligent infliction of emotional distress, and produced evidence at trial conclusively establishing that the construction company was negligent in allowing the air conditioning equipment to fall. Which of the following is the construction company's best defense against this action? Answers: A: The mother was outside the zone of danger. B: The mother did not suffer a physical impact to her person. C: The son did not suffer an injury. D: The construction company's actions were not extreme and outrageous.

Answer choice C is correct. In order for a plaintiff to recover under a theory of negligent infliction of emotional distress, the plaintiff must suffer physical symptoms. The plaintiff in this action, the boy, did not suffer any ill effects as a consequence of the construction company employee's negligent handling of the equipment. Because nominal damages are not awarded in a negligence action, the son's lack of injury is the company's best defense. Answer choices A and B are incorrect because they relate to the mother, who is not a party to the action. Answer choice D is incorrect because "extreme and outrageous conduct" is an element of intentional infliction of emotional distress, not negligent infliction of emotional distress.

An art thief planned to steal a three-dimensional piece of hanging installation art from a modern art museum. Her plan involved dressing up like a museum docent, temporarily closing off the room that contained the installation art, and in the one minute it took for the security camera to visually sweep the rest of the room, to hide the installation art in a covered garbage can. Before the police or museum security would think to search a garbage can, the art thief would then disguise herself as part of the janitorial crew and remove the garbage can containing the installation art from the museum. However, on the day she planned to execute her heist, the art thief found out that the museum had recently installed hidden GPS tracking devices on all of the art pieces in the museum. The art thief decided to postpone her heist until she could figure out how to disable the GPS device and avoid getting caught. A few days later, the art thief was arrested by the police based on security footage from the museum that showed the art thief engaged in suspicious activities like stealing docent and janitorial uniforms and timing the security cameras. Can the art thief be charged with attempted larceny of the artwork? Answers: A: No, because the element of "asportation" was not satisfied. B: No, because she abandoned the crime before it occurred. C: Yes, because she took a substantial step towards stealing the artwork. D: Yes, because attempted larceny is a general intent crime.

Answer choice C is correct. In order to be guilty of attempt, a defendant must take a substantial step towards the commission of the offense with the specific intent to commit the crime. Here, the art thief took a substantial step towards the commission of larceny of the artwork when she stole the uniforms. Consequently, she can be charged with the offense of attempted larceny of the artwork. Answer choice A is incorrect because, while asportation of the object (i.e., the moving and carrying away of the object) is an element of larceny, for the purposes of attempted larceny the person who intends to perpetrate the larceny must merely take a substantial step towards the commission of a crime. The art thief's intent to move and carry away the artwork is sufficient. Answer choice B is incorrect. At common law, once the defendant has taken a substantial step toward the commission of the offense, the defendant may not legally abandon the attempt to commit the crime because of a change of heart. While some states do recognize voluntary abandonment as a defense to attempt, abandonment is not voluntary if it is motivated only by a desire to avoid detection. The art thief's abandonment was not voluntary because it was motivated by her fear of getting caught due to the GPS tracking device. Answer choice D is incorrect because attempted larceny is a specific, not general intent crime.

An 18-year-old defendant was arrested for armed robbery. After being Mirandized, the defendant asked for a lawyer. The police did nothing about this request, but instead contacted the defendant's parents who came to the police station where the defendant was being held. The police placed the defendant and his two parents in a bugged room in the hopes that the defendant would incriminate himself. The defendant confessed to his parents that he committed the armed robbery. Learning that the prosecution now plans to use the confession at the defendant's trial, the defendant's attorney has filed a motion to suppress this confession as a violation of the defendant's Fifth Amendment right to counsel. Of the following, which represents the best factual support for the defense's motion? Answers: A: The defendant asked for an attorney after being Mirandized. B: The defendant did not know that the room was bugged. C: The police placed the defendant and his parents in a room in the hopes that the defendant would incriminate himself. D: The police took unfair advantage of the defendant's voluntary statements to his parents.

Answer choice C is correct. Once the right to counsel is invoked, the defendant may not be subjected to further interrogation without counsel present. Interrogation refers not only to express questioning, but also to any words or actions that the police know or should know are likely to elicit an incriminating response. Here, the police appear to have arranged for the defendant to meet with his parents in an effort to obtain an incriminating statement. Therefore, this would be the defendant's best argument for excluding the statement. Answer choice A is incorrect because the police's failure to comply with the student's request for an attorney, by itself, will not make the statement inadmissible. The statement is inadmissible only if it was the product of police interrogation. Answer choice B is incorrect. After a defendant who is in police custody has requested an attorney, the focus is on whether the defendant's statement was a product of police interrogation. Therefore, the determinative issue is whether the police knew or should have known that their words or actions were likely to elicit an incriminating response, and not the defendant's knowledge of whether he was being recorded. Answer choice D is incorrect because volunteered statements are not protected by Miranda, as they are, by definition, not the product of interrogation. The defendant should instead argue that his statements were not voluntary, and that he was coerced into his statements as a result of police interrogation.

A plaintiff, a citizen of State X, sued a defendant, a citizen of State Y, for negligence in federal district court in State X under diversity jurisdiction, in connection with an automobile accident that occurred in State Y. The defendant has had no contacts with State X. The plaintiff personally served the defendant with a summons and complaint at his home in State Y. The defendant's first response to the complaint was an answer that specifically denied the plaintiff's claims but omitted the defense of lack of personal jurisdiction. Fifteen days after serving the answer on the plaintiff, the defendant amended the answer to include the defense of lack of personal jurisdiction without asking leave of the court. Which of the following statements is most accurate regarding the defendant's actions? A: The defense of lack of personal jurisdiction was permanently waived when the defendant's answer failed to include it. B: The defense of lack of personal jurisdiction has been waived unless the court subsequently grants the defendant leave to amend his answer. C: The defense of lack of personal jurisdiction was not waived and may be asserted by the defendant. D: The defense of lack of personal jurisdiction can never be waived and may always be asserted by a defendant.

Answer choice C is correct. Pursuant to Rule 15(a)(1)(a), a party may amend its pleading once as a matter of course within 21 days after serving it. Under Rule 12(h)(1)(B), a party waives the defense of lack of personal jurisdiction by failing to include it in a pre-answer motion, a responsive pleading, or in an amendment allowed by Rule 15(a)(1), as a matter of course. Here, the defendant included the defense of lack of personal jurisdiction in an amendment made within 21 days after serving its answer. Accordingly, the defense was not waived and may be asserted by the defendant in the case. Answer choice A is incorrect. The failure to include the defense of lack of personal jurisdiction in the defendant's answer would have waived the defense, but the defendant subsequently amended his answer within 21 days of serving it to include the defense. Answer choice B is incorrect. Because the defendant amended the answer within 21 days of serving it, leave of court is not required to amend, pursuant to Rule 15(a)(1)(a). Answer choice D is incorrect. The defense of lack of personal jurisdiction (unlike lack of subject matter jurisdiction) can be waived, as discussed above.

Relying on its Commerce Clause power, Congress enacted a statute that criminalized the knowing possession of sexually explicit visual displays of a minor. Congress adduced facts that possession of such displays substantially affected interstate commerce. Simulated displays, or those that use youthful looking adults or computer graphic techniques instead of minors, are not prohibited. Amalgamate displays, or those that contain actual minors but with the sexually explicit aspect created by using youthful looking adults or computer graphic techniques, are prohibited. The definition of the crime is neither vague nor overbroad. A defendant who possessed amalgamated displays in his home and did not sell or otherwise transfer them was convicted under this statute. He has challenged his conviction, contending that the statute is unconstitutional. Should an appellate court uphold his conviction? Answers: A: No, because the statute exceeds congressional power under the Commerce Clause by regulating the possession of material that has not entered into the stream of commerce. B: No, because the statute violates the defendant's privacy interest which is protected by the Fifth Amendment Due Process Clause. C: Yes, because the statute does not violate the First Amendment Free Speech Clause. D: Yes, because there is no constitutional right to possess pornography.

Answer choice C is correct. The First Amendment does not protect child pornography. Because of the government's compelling interest in protecting minor children from exploitation, the sale, distribution, and even private possession of child pornography may be prohibited, even if the material would not be obscene if it involved adults. Simulated displays may not be banned, but amalgamated displays may be banned. Accordingly, the statute passes constitutional muster. Answer choice A is incorrect because Congress has broad power under the Commerce Clause to regulate interstate commerce. This power extends to privately produced and consumed items when those items have a substantial impact on interstate commerce, provided that Congress has adduced facts that establish this impact. Answer choice B is incorrect because Congress may criminalize the possession of child pornography without violating an individual's due process rights. Answer choice D is incorrect because there is a fundamental right to possess obscene materials, including pornography that does not include children, in the privacy of one's home.

The Judiciary Committee of the U.S. House of Representatives initiated impeachment proceedings against a federal district court judge. The President, a lifelong friend of the judge, considered the grounds for impeachment that were being discussed to be politically motivated and without substantial merit. Prior to any hearing on matter by the House committee, the president pardoned the judge. What effect does this pardon have on the impeachment proceedings against the judge? Answers: A: The proceedings must stop, because the President's power to pardon is plenary. B: The proceedings must stop, because the President acted in good faith in granting the pardon. C: The pardon has no effect on the proceedings because a President's power to pardon does not extend to impeachment. D: The pardon has no effect on the proceedings because a President may not pardon a person until that person has been convicted.

Answer choice C is correct. The President's power to pardon does not extend to impeachment of a federal official. Answer choice A is incorrect. While the President's power to pardon with regard to federal crimes is often referred to as plenary, it does not extend to impeachment of a federal official. Answer choice B is incorrect because there is no good faith restriction imposed by the Constitution on the President's exercise of the power to pardon. Answer choice D is incorrect because, while a President may not pardon a person with respect to future acts, a President may pardon a person at any time after the offense has been committed.

A plaintiff sued a defendant for breach of contract. The plaintiff claimed that she had ordered 250 widgets from the defendant and that the defendant had only delivered 150 widgets. In his answer, the defendant claimed that the plaintiff had only ordered 150 widgets. On the witness stand, the defendant testified that the plaintiff had ordered the widgets by filling out a purchase order form provided by the defendant. The defendant's counsel then asked the defendant "What number did the plaintiff fill in on the form for the number of widgets ordered?" The defendant answered "150." The plaintiff objected to the question, and asked the judge to strike both the question and the answer. How should the judge rule? Answers: A: Deny the motion, as the statement was made by an opposing party. B: Deny the motion, as the statement falls under the exception to the hearsay rule for records of a regularly recorded activity. C: Grant the motion, as the best evidence rule applies. D: Grant the motion because the statement constitutes hearsay not within any exception.

Answer choice C is correct. The best evidence rule requires that the original document be produced in order to prove the contents of that document, when the contents of the document are at issue. Here, the defendant is giving testimony about the contents of a written document, the contract, which are directly at issue in the case. Further, the defendant has not produced the original document or shown an excuse for nonproduction. Answer choice A is incorrect because, even though the statement would be a statement by a party to the litigation, the best evidence rule requires that to prove the content of a writing, the original writing is generally required. Note that if the contents of the document had been admitted by the plaintiff in another writing, the best evidence rule would not apply. Answer choice B is incorrect because even though the document itself might possibly come under the business records exception to the hearsay rule, the best evidence rule would still apply. Remember that a piece of evidence that is admissible under one rule (such as a hearsay exception) may still be inadmissible for another reason. Answer choice D is incorrect, as the statement was one made by an opposing party, which Rule 801(d)(2)(A) expressly defines as nonhearsay. The motion should be granted under the best evidence rule.

A boat owner initiated a products liability action against the manufacturer of the boat's engine. The owner alleged that the engine manufacturer failed to warn the owner about the proper operation of a switch on the engine, and that improper operation of the switch caused the owner's injuries. The boat owner offered evidence that the manufacturer had begun including a written warning for the switch for all boats manufactured beginning in the year after the owner's boat was manufactured. The owner had owned his boat for five years prior to his injury. Is this evidence admissible? Answers: A: No, because evidence of the manufacturer's warning is inadmissible as a remedial measure. B: No, because the remedial measures exclusion is limited to negligence cases. C: Yes, because the manufacturer began providing the warning before the boat owner's accident. D: Yes, because evidence of the need for a warning is not subject to the remedial measures exclusion.

Answer choice C is correct. The court must exclude evidence of subsequent remedial measures undertaken by a defendant after the event that caused a plaintiff's alleged injury or harm. Here, the manufacturer began providing a warning before the boat owner's accident. For this reason, answer choice A is incorrect. Answer choice B is incorrect because the exclusion for remedial measures applies to products liability actions as well as negligence actions. Answer choice D is incorrect because the remedial measures exclusion does apply to evidence offered to prove the need for a warning.

A man placed an advertisement on an internet website offering to sell a rare coin in return for $50,000. A woman responded to the advertisement, met the man, inspected the coin, and offered to buy it. The man told her that others had expressed an interest in the coin, but that she could have it if she paid a $15,000 cash deposit within one day, with the balance paid by the end of the week. The woman came back the next day and gave the $15,000 to the man, who promised to give her the coin later that week in return for the balance of the purchase price. The man took the $15,000 and promptly flew out of state with the coin, which he never intended to sell in the first place. With which of the following crimes can the man properly be charged? Answers: A: Larceny by trick B: Embezzlement C: False pretenses D: None of the above

Answer choice C is correct. The crime of false pretenses requires obtaining title to the property (including money) of another person through the reliance of that person on a known false representation of a material past or present fact, when the representation is made with the intent to defraud. Here, the woman transferred title to her $15,000 to the man, relying on his false representations, which were made with the intent to defraud her. Accordingly, the man can properly be charged with the crime of false pretenses. Answer choice A is incorrect. Larceny by trick requires that the defendant obtain possession of, but not title to the property owned by another. Here, the woman intended to pass title to the money, not merely temporary possession, to the man. Accordingly, larceny by trick does not apply. Answer choice B is incorrect. Embezzlement requires fraudulent conversion of the property of another by a person who is in lawful possession of the property. Here, the man was not in lawful possession of the property, since he acquired it by fraud. Therefore, embezzlement does not apply. Answer choice D is incorrect because the man can be charged with false pretenses.

A single woman desperately wanted to have a baby, so she utilized the services of a sperm bank to get pregnant. The woman had a difficult pregnancy, and she suffered from postpartum psychosis after the birth of her baby son. She began seeing a psychiatrist, but rather than work through her postpartum psychosis, she spent the majority of her sessions talking about how she was convinced that the sperm bank had used her as part of a top-secret government experiment, and as a result of the experiment, she had given birth to Hitler's spawn. Although the woman had yet to harm her son, she grew more and more adamant in her belief that Hitler was the father of her son. Believing that she needed to save the world from another Hitler, she drowned her baby son. The woman was charged with murder, and her defense attorney raised an insanity defense. During the trial, the psychiatrist testified that the woman would not have killed her baby in the absence of postpartum psychosis, and that the woman did not think it was immoral to drown her son because she believed he was Hitler's spawn. The jurisdiction in which the court resides has adopted the M'Naghten test for insanity. What is the woman's strongest argument in support of an insanity defense? Answers: A: The woman did not know the quality of the act she was performing. B: The woman's acts were the product of a mental disease. C: The woman did not know that her act was morally wrong. D: The woman did not know the nature of the act she was performing.

Answer choice C is correct. Under the M'Naghten test, the defendant is not guilty if, because of a defect of reason due to a mental disease, the defendant did not know either the nature and quality of the act, or the wrongfulness of the act. Here, the woman suffered from postpartum psychosis, and she did not understand that it was morally wrong to drown her son. Answer choice A and D are incorrect because neither of these choices are supported by the facts. The woman understood the nature and quality of her actions. She understood that her actions would result in the death of a human being; in fact that was the purpose of her actions--to drown her son. Additionally, neither identifies the central issue, which is that without knowing that the act is wrong, the woman could not have formed the requisite criminal intent. Answer choice B is incorrect because it states the test for insanity under the Durham rule.

A vendor, in a signed writing, contracted with a manufacturer for the sale to the vendor of 100 identical fine china tea sets, 25 sets to be delivered on April 15, May 15, June 15, and July 15. The agreement did not specify the time or place of payment. Which of the following statements regarding payment under the contract is correct? Answers: A: The vendor must pay the manufacturer upon delivery of all 100 sets. B: The vendor must pay the contract price for the first 25 sets within a reasonable time after delivery on April 15. C: The vendor must pay the contract price for the first 25 sets when they are delivered on April 15. D: The vendor must pay the contract price for all 100 sets when the first 25 sets are delivered on April 15.

Answer choice C is correct. Under the UCC, an installment contract is defined as one in which the goods are to be delivered in multiple shipments, and each shipment is to be separately accepted by the buyer. Parties cannot vary or contract out of this definition under the code. Payment by the buyer is due upon each delivery, unless the price cannot be apportioned. This agreement is an installment contract because the china tea sets are to be delivered in numerous shipments. Although the contract is silent as to when payment is due, the UCC provisions make payment due upon each delivery. Answer choice A is incorrect because here, the price can be apportioned, and thus the price is due upon each delivery. Answer choice B is incorrect because here, the payments can be apportioned and are due upon each delivery according to the UCC, not within a reasonable time after delivery. Answer choice D is incorrect because the full contract price for the 100 sets is not due upon delivery of the first 25 china tea sets. The vendor is required to pay the contract price only for the 25 sets delivered.

An auto mechanic borrowed money to buy an auto repair garage. As security for repayment of the loan, the mechanic transferred ownership of the garage to a title company pursuant to a deed of trust. Under the terms of the deed of trust, upon notification by the lender that the auto mechanic had defaulted on the loan, the title company was authorized to transfer title to the lender if after three months the auto mechanic had not redeemed the property. This jurisdiction recognizes a deed of trust as an acceptable form for a security interest in real property. One year later, the auto mechanic defaulted on the loan. The lender notified the title company of the default. After three months, the title company transferred ownership of the garage to the lender pursuant to the terms of the deed of trust. The following month, the auto mechanic attempted to repay the full amount of the loan obligation. The lender rejected this payment. The auto mechanic has filed suit to regain title to the auto repair garage. How should the court rule? Answers: A: In favor of the lender, because the auto mechanic did not redeem the garage within three months after the lender notified the title company. B: In favor of the lender, because a deed may be transferred in lieu of foreclosure. C: In favor of the auto mechanic, because the clause in the deed that limited the time to redeem the garage is invalid. D: In favor of the auto mechanic, because he used the money from the loan to purchase the garage.

Answer choice C is correct. Until property that is subject to a mortgage or a deed of trust is foreclosed upon by sale, the borrower has a right to redeem the property by satisfying the obligation for which the property serves as security. While this right may be waived after the borrower has defaulted, the terms of the mortgage or deed of trust cannot waive this right as a matter of course. Consequently, because the clause in the deed of trust in question is invalid and the court should permit the auto mechanic to regain title to the garage by paying the loan obligation in full. Answer choice A is incorrect because, although the auto mechanic did not comply with the terms of the deed of trust, the term that required redemption of the garage within three month was invalid as a clog on his equitable right to redeem the property prior to a foreclosure sale. Answer choice B is incorrect because, while after default a borrower may agree to transfer property that served as security for a loan to the lender in lieu of foreclosure, here the provision for the transfer of the deed in lieu of foreclosure was contained in the deed of trust itself. Therefore, it constitutes an unenforceable clog on the mechanic's equitable right of redemption. Answer choice D is incorrect because the equitable right of redemption is not limited to purchase-money mortgages or deeds of trust.

An engineer at a technology company signed a non-compete agreement preventing him from discussing confidential matters with specific named competitors. One year later, the company learned that the engineer had been working with a major competitor as a contractor and had been using confidential information acquired during his employment with the technology company. In order to immediately prevent the engineer from using information in violation of the non-compete agreement, the technology company properly filed a complaint against the engineer in federal court, alleging a violation of the agreement and seeking an injunction that would prevent the engineer from continuing to work with the competitor. Over the engineer's objection, the court issued a temporary restraining order that held the non-compete agreement to be valid and enforceable, and prohibited the engineer from working with the competitor. The court further stated that the restraining order would remain in effect for 21 days, with a full hearing on a preliminary injunction scheduled for the day the temporary restraining order expired. Twenty days later, the engineer resumed working with the competitor. Has the engineer violated the temporary restraining order? Answers: A: Yes, because the hearing on the matter was scheduled for the next day. B: Yes, because the engineer violated the non-compete agreement. C: No, because the court does not have the authority to modify the timeframe for a temporary restraining order. D: No, because the temporary restraining order was no longer in effect.

Answer choice D is correct. A temporary restraining order (TRO) preserves the status quo of the parties until there is a full hearing on the application for a preliminary injunction. This temporary injunction remains in effect for an amount of days to be set by the court, but no longer than 14 days unless good cause exists or by consent of the adversary. Here, the court could determine the applicable period, but could not extend that period beyond 14 days absent good cause or consent of the adversary. The facts indicate that the engineer (the adversarial party) objected to the TRO, and there was no good cause shown other than the urgency of any normal injunction. Therefore, the customary 14-day period would apply. Answer choice A is incorrect because although a hearing was scheduled, the TRO itself was no longer in effect, as explained above. Answer choice B is incorrect because the violation of the non-compete agreement is another matter, independent of the violation of the TRO. Answer choice C is incorrect because the court can set a different time period for the TRO, but cannot extend it beyond 14 days absent good cause or the consent of the other party.

A paving company entered into a contract with a real estate developer to repave a large parking lot in the developer's shopping center. Since the paving company wanted to establish a good reputation in the market, it discounted its price. The paving company expected to make $20,000 in profit on the contract. Midway through the project, the developer notified the paving company to cease work; the paving company immediately complied. At the time of the notice, the paving company had incurred costs of $200,000. The cost of hiring another contractor to perform the same work would have been $225,000. The paving work performed had increased the value of the shopping center by $100,000. The paving company filed a lawsuit against the developer. The fact finder determined that the developer's repudiation of the contract was without justification. What is the maximum amount of damages the paving company can be awarded? Answers: A: $100,000, the amount by which the value of the shopping center was increased. B: $200,000, the costs incurred by the paving company. C: $220,000, the developer's expectancy damages. D: $225,000, the amount that the developer would have had to pay for the portion of the paving job that was completed.

Answer choice D is correct. Although expectancy damages normally are awarded in a breach-of-contract action, restitutionary damages are permitted in cases where the nonbreaching party has partially performed a below-market-price contract. Otherwise, the breaching party would profit from its breach. Consequently, the paving company may recover the benefit conferred upon the developer as measured by the amount the developer would have had to pay to secure the same performance as that rendered by the paving company. Answer choice A is incorrect because, although the increase in the defendant's property value due to the plaintiff's performance is one measure of restitutionary damages, that amount is less than the paving company's restitutionary damages as measured by the cost to the developer of obtaining the same performance rendered by the paving company. Answer choice B is incorrect because the paving company's reliance damages are exceeded by both its expectancy and restitutionary damages. Answer choice C is incorrect because the paving company's expectancy damages, based on the cost incurred by the company plus its profit on the contract, are less than the company's restitutionary damages.

A daughter successfully petitioned a court to have her father declared incompetent to manage his affairs and to have herself appointed as guardian of his property. Subsequently, the father ordered furniture totaling $3,500 from a local store. The store, unaware of the guardianship and not otherwise having a reason from the father's behavior to learn of his incompetency, delivered the furniture to the father's residence where he received and accepted it. The next day a flood destroyed the furniture before the daughter had the opportunity to contact the store. Is the store entitled to enforce the contract for the sale of the furniture? Answers: A: Yes, because the risk of loss had passed to the father, as buyer of the furniture, upon its delivery. B: Yes, because the store was unaware of the guardianship and the father's incompetency. C: No, because the daughter did not have the opportunity to contact the store. D: No, because the father had been adjudicated incompetent.

Answer choice D is correct. An individual who is the subject of a court-ordered guardianship over that individual's property lacks the capacity to enter into a contract. Consequently, any contract purportedly entered into by such an individual is void. Since the father was under a court-ordered guardianship, his contract to purchase the furniture was void. (Note: If the furniture is a necessity, the store may be able to recover the furniture's reasonable value in restitution under a quasi-contract action, but cannot enforce the contract of sale between the store and the father.) Answer choice A is incorrect because, although where a contract for the sale of goods exists the delivery of the goods by the seller to the buyer generally shifts the risk of loss to the buyer, here the purported contract entered into by the father for the purchase of the furniture was void. Answer choice B is incorrect because, where a guardianship over an individual's property has been recognized by a court, the seller's lack of actual knowledge of the guardianship or the individual's inability to contract is irrelevant. Answer choice C is incorrect because, since the father's contract with the store was void rather than merely voidable, the daughter's lack of opportunity to contact the store is irrelevant.

A husband and his friend agreed to kill the husband's wife, who had been unfaithful, and split the insurance proceeds the man received following the wife's death. The friend helped the husband obtain a handgun to commit the murder, but on the night of the planned murder, the friend decided that she could not go through with it. Afraid that the husband would kill her if she went to the police, she did not contact them. Instead, she called the husband and told him that she could not go through with the crime and urged him not to kill his wife. The husband ignored her and went through with the killing. The husband and friend were subsequently arrested and the friend was charged with conspiracy to commit murder. Should the friend be convicted? Answers: A: No, because she successfully withdrew from the conspiracy. B: No, because at the time of the murder, the friend lacked the intent for the wife to be killed. C: Yes, because the friend committed an overt act in furtherance of the conspiracy. D: Yes, because withdrawal is not a valid defense to conspiracy.

Answer choice D is correct. At common law, withdrawal is not a defense to conspiracy because the conspiracy is complete as soon as the parties enter into the agreement. Accordingly, as soon as the man and his friend agreed to kill the man's wife, the conspiracy was complete. Remember that, unless the fact pattern specifically indicates otherwise, the common-law rule for conspiracy applies on the MBE. Answer choice A is therefore incorrect. Answer choice B is incorrect because conspiracy requires that the conspirator had the intent to agree and the intent to commit the criminal objective at the time of the agreement, not at the time that the criminal objective is accomplished. Answer choice C is incorrect because at common law, no overt act is required for the conspiracy to be complete. All that is required is (i) an agreement, (ii) between two or more persons, (iii) to accomplish an unlawful purpose, (iv) with the intent to accomplish that purpose.

After a prominent senator is accused of accepting bribes from commercial gaming lobbyists in exchange for a promise to vote in favor of land-based casinos in a state, a Congressional committee is assigned to investigate the alleged misconduct. Within the course of the investigation, the committee seeks to question Senator X, a fellow senator from the same state as the accused senator. Senator X is subpoenaed to appear before the Congressional committee without the presence of counsel to testify on whether he ever saw the accused senator accept money from a gaming lobbyist. Senator X fails to appear before the Congressional committee and is immediately cited for contempt. Is the citation of Senator X for contempt valid? Answers: A: Yes, because Senator X did not appear before the committee despite receiving the subpoena. B: Yes, because Congress has the right to investigate any matter within a legitimate legislative sphere. C: No, because the committee exceeded its investigatory power. D: No, because Senator X did not receive procedural due process.

Answer choice D is correct. Congress does not have an express power to investigate, but the Necessary and Proper clause allows Congress broad authority to conduct investigations incident to its power to legislate. While a subpoenaed witness who fails to appear before Congress or refuses to answer questions may be cited for contempt, the witness is entitled to certain procedural due process rights, including the presence of counsel. Here, because Senator X did not receive procedural due process, the citation for contempt would be invalidated. Answer choice A is incorrect because although Senator X could be cited for contempt if the other requirements (such as procedural due process and the privilege against self-incrimination) were met, the requirements of due process are not met here in light of the lack of counsel. Answer choice B is incorrect because while Congress can investigate all matters within a legitimate legislative sphere, which would include the possible acceptance of bribes in exchange for votes, any subpoenaed witness who fails to appear would still be entitled to procedural due process. Answer choice C is incorrect because the committee did not exceed its investigatory power, as whether the senator accepted bribes in exchange for votes related to commercial gaming would fall into the legislative sphere.

Investors brought an action under federal law for fraud in the sale of securities against an investment company. The action was dismissed with prejudice by the federal district court because it was not timely filed. The investors did not appeal this dismissal. Congress then passed legislation permitting the investors to reinstate this action. The investors petitioned the district court for reinstatement of their action. Of the following, which would serve as the best ground for the investment company to challenge the constitutionality of this law? Answers: A: Bill of Attainder Clause of Article I, Section 9 of the U.S. Constitution B: Due Process Clause of the Fourteenth Amendment C: Ex Post Facto Clause of Article I, Section 9 of the U.S. Constitution D: Separation of powers doctrine

Answer choice D is correct. Congress may not reinstate the right to bring a legal action after the judgment in the action has become final. Requiring a federal court to do so violates the separation of powers doctrine. Once a judicial decision becomes the final word of the federal judiciary with regard to a particular case or controversy, Congress may not declare by retroactive legislation that the law applicable to that particular case was different from what the courts said it was. Answer choice A is incorrect. A bill of attainder is a legislative act that declares a person or group of persons guilty of some crime and punishes them without a trial. Although the federal government as well as the states are constitutionally prohibited from enacting such legislation, the statute in question is not a bill of attainder because it did not declare the defendant investment company guilty of a crime and punish it without a trial. Answer choice B is incorrect because the Due Process Clause of the Fifth Amendment, not the Fourteenth Amendment, applies to the federal government. Answer choice C is incorrect. Although the constitutional prohibition on ex post facto laws does apply to the federal government as well as the states, the statute in question is not an ex post facto law because it did not constitute a retroactive change in a criminal or penal law (or a civil law with an overriding punitive effect). It merely attempted to permit the reinstatement of the investors' lawsuit.

A man owned a 25-acre tract of land. He conveyed 20 of the 25 acres to a developer by warranty deed and continues to live on the five-acre portion he retained. The deed to the 20-acre tract was promptly recorded and contained the following: "It is a condition of this deed that all owners, their heirs and assigns, of any portion of the 20-acre tract shall use the land for single-family residences only." The applicable zoning ordinance allows for single and multi-family homes in this area. The developer fully developed the tract into a residential subdivision consisting of 20 lots with a single-family home on each lot. The lots were subsequently sold and the deed to each lot referenced the quoted provision. A woman purchased one of the lots and decided to build an addition to the house. The woman plans to build an entirely separate apartment and rent it to college students. A nosy neighbor in an adjoining subdivision opposes this development because she does not want rowdy college students driving through the neighborhood. Can the neighbor prevent the woman from building the apartment? Answers: A: Yes, because the original parties intended for the rights and duties to run with the land. B: Yes, because the restriction is valid under the common-law Rule Against Perpetuities. C: No, because the zoning ordinance allows for multi-family homes as well as single-family homes. D: No, because the neighbor does not have the right to enforce the restriction.

Answer choice D is correct. Equitable servitudes are covenants about land use that are enforced at equity by injunction. For a servitude to be enforced at equity, it must be in writing and meet the following requirements: (i) there must be intent for the restriction to be enforceable by and against successors in interest, (ii) the servitude must touch and concern the land, and (iii) the person against whom the servitude is to be enforced must have notice (whether actual, record, or inquiry notice) of the servitude. In this case, there is a valid equitable servitude on the 20-acre subdivision. The restriction is contained in the deed between the original parties. The phrase "all owners, their heirs and assigns" shows the parties' intent that the benefits and burdens run with the land. The restriction touches and concerns the land because it affects the owners as property owners, not merely individuals. Finally, subsequent purchasers had notice of the restriction in their deeds. However, the neighbor cannot enforce the equitable servitude because she does not own property in that subdivision. The benefit of enforcing an equitable servitude is held only by successors in interest to the original deed. Answer choice A is incorrect. Although the original parties did intend for the rights and duties to run with the land by their language, "all owners, heirs and assigns," this would not give the neighbor, who does not own property in the subdivision, the right to prevent the woman from building the apartment. Answer choice B is incorrect. Equitable servitudes are not affected by the Rule Against Perpetuities. However, the mere fact that the restriction is not subject to the Rule does not mean the neighbor has a right to enforce the restriction. Answer choice C is incorrect because the equitable servitude prevails over the zoning ordinance in terms of its limitations. A zoning restriction establishes a "ceiling" beyond which a private covenant cannot go, but it does not set a "floor" below which a private covenant cannot go. Another owner in the same subdivision could enforce the equitable servitude against the woman. However, the neighbor cannot.

Two farmers who owned adjoining farms entered into a written agreement, promising that each farmer, as well as "his heirs and assigns," would not use chemical pesticides or herbicides in farming the land. This agreement was promptly and properly recorded in the land records for the county in which the farms were located. One farmer died and left his farm to his only child, a son, for life and then to his grandson. The other farmer died and left half of his farm to one of his daughters and the other half to his other daughter. The son began to use chemical pesticides and herbicides on the farm that he inherited. One of the farmer's daughters brought an action seeking an injunction against the son to prevent his continued use of chemical pesticides and herbicides on his farm. Is the daughter's action likely to succeed? Answers: A: No, because the son holds only a life estate interest in his farm. B: No, because the daughter owns only one-half of the farm that was formerly owned by her father. C: Yes, because both the daughter and the son are in vertical privity with their respective predecessors-in-interest. D: Yes, because the two original farmers created an equitable servitude that is enforceable by the daughter against the son.

Answer choice D is correct. Equitable servitudes are covenants about land use that are enforced in equity by injunction. For an equitable servitude to exist, there must be a written agreement that reflects an intent for the restriction to be enforceable by and against successors in interest. In addition, the restriction must touch and concern the land, and the person against whom the servitude is to be enforced must have notice of the restriction. In this case, the daughter is seeking to obtain an injunction against the son. Thus, the daughter must establish that the agreement between the two farmers (the parties' fathers) created an equitable servitude. Here, each farmer bound his heirs and assigns to the written agreement, the prohibition on the use of chemical pesticides and herbicides directly related to the use of the land for farming, and the agreement was recorded, which provided the son with constructive notice of the agreement. Therefore, the requirements for enforcing an equitable servitude have been met, and the daughter should be granted an injunction against the son. Answer choice A is incorrect. Although the type of property interest held by the son in the farm (i.e., a life estate) means that vertical privity does not exist between the son and his father, the existence of vertical privity is not required for an equitable servitude to be enforceable. Answer choice B is incorrect because vertical privity does exist between the daughter and her father. Even though she received only one-half of the farm owned by her father in geographic terms, she took the full durational interest held by her father. More importantly, whether there is vertical privity between the daughter and her father is irrelevant for the purposes of enforcing an equitable servitude. Answer choice C is incorrect because vertical privity does not exist between the son and his father because the son did not take the full durational interest held by his father. Moreover, whether there is vertical privity between the daughter and the son and their fathers is irrelevant for the purposes of enforcing an equitable servitude.

One afternoon, while driving a co-worker home, a man suffered a seizure and suddenly lost consciousness. While he was unconscious, he drove off the road and slammed his car into a brick wall. As a result of the impact, the man's passenger was killed. The man was a diagnosed epileptic, had suffered several seizures in the past, and recently had voluntarily stopped taking a medication intended to control recurrences. The man was charged with common-law involuntary manslaughter, and was convicted. On appeal, the defendant contends that his conviction should be reversed. How should the appellate court rule on the appeal? Answers: A: Reverse the conviction, because the man's reckless actions did not manifest extreme indifference to the value of human life. B: Reverse the conviction, because the man did not commit an act sufficient to satisfy the actus reus requirement. C: Affirm the conviction, because the man voluntarily stopped taking his anti-seizure medication. D: Affirm the conviction, because the man engaged in reckless and criminally negligent behavior.

Answer choice D is correct. If an epileptic knows of the possibility of a seizure and engages in the voluntary act of driving a car, has a seizure while driving, and causes a fatal accident, the epileptic is criminally responsible. While the man's act of driving into the brick wall was performed while experiencing a seizure, and thus was not volitional and does not satisfy the actus reus requirement, the man's act of driving the car while knowingly suffering from the condition is considered a sufficient actus reus. Answer choice A is incorrect because it states the requirements for a depraved heart murder conviction; here, the man was convicted of involuntary manslaughter. Answer choice B is incorrect because the man's act of driving the car while knowingly suffering from his condition is considered a sufficient actus reus. Answer choice C is incorrect because, while ceasing to take anti-seizure medication may have increased the man's risk of suffering a seizure, it cannot, standing alone, support criminal liability. The negligent act was driving the car after failing to take this medication.

A grocery store and a farmer entered into a valid contract for 1,200 bottles of Grade A maple syrup, to be delivered over the course of a year in twelve equal installments of 100 bottles, delivered on or before the last day of each month, beginning in January and ending in December. From January to March the farmer delivered three shipments of conforming Grade A maple syrup. In April, on the last day of the month, the farmer delivered to the store 97 bottles of Grade A maple syrup. The grocery store rejected the syrup, and informed the farmer that the contract between them was canceled. Which of the following is a correct statement of the grocery store's actions? Answers: A: The store's rejection of the 97 bottles of Grade A maple syrup and cancellation of the contract were proper. B: The store's rejection of the 97 bottles of Grade A maple syrup was proper, but the store's cancellation of the contract was improper. C: The store's rejection of the 97 bottles of Grade A maple syrup was improper, but the store's cancellation of the contract was proper. D: The store's rejection of the 97 bottles of Grade A maple syrup and cancellation of the contract were improper.

Answer choice D is correct. In general, the UCC requires perfect tender of goods, and substantial performance will not suffice. However, the UCC does permit substantial performance with regard to an installment contract. In addition, when there is a nonconforming tender or a tender of nonconforming goods under one segment of an installment contract, the buyer may cancel the contract only if the nonconformity substantially impairs the value of the entire contract to the buyer. Here, the farmer made a tender of three fewer bottles of syrup than called for under the contract. However, shipping 97 out of 100 bottles of maple syrup likely qualifies as substantial performance, and the grocery store was not entitled to reject the shipment. Further, the shortfall of three bottles likely does not substantially impair the value of the entire contract to the grocery store. Answer choice A is incorrect. Because the three-bottle shortfall likely did not substantially impair the value of the April installment, the grocery store's rejection of the farmer's tender of 97 bottles was improper. In addition, since this shortfall likely does not substantially impair the value of the entire contract to the grocery store, the store's cancellation of the entire contract was improper. Answer choices B and C are incorrect for the same reason.

A driver from State A was involved in a major car accident in State B with a bus driver from State C and a livery driver from State D. The driver suffered serious injuries in the accident, resulting in $150,000 in medical bills. The driver sought to bring a diversity action in federal district court based on negligence against the bus driver and the livery driver. Assume each state only has one federal district court, and each court has jurisdiction over all of the parties. Where is venue proper? Answers: A: Either State C or State D B: Either State A or State B C: State A only D: State B only

Answer choice D is correct. In general, venue in a federal civil action is proper in only one of the following judicial districts: (i) a judicial district in which any defendant resides, if all defendants reside in the same state in which the district is located, or (ii) a judicial district in which a "substantial part of the events or omissions" on which the claim is based occurred, or where a "substantial part of the property" that is the subject of the action is located. In this case, the car accident (the event) occurred in State B. The driver's claim arose out of the car accident. Therefore, venue is proper in State B. Answer choice A is incorrect. Venue is proper in the judicial district in which any defendant resides only if all defendants reside in the same state in which the district is located. In this case, the defendants, the bus driver and the livery driver, are from different states. Thus, venue is not proper in State C or State D. Answer choice A is incorrect. The federal rules for venue do not look to the place of the plaintiff's residence to determine where venue is proper. Here, the fact that the driver is from State A is not relevant in deciding proper venue for the driver's action. Answer choice C is incorrect because the judicial district where the driver resides is not a proper venue.

As a car came to a stop at a stop sign, a pedestrian watched as a box fell off the roof of the car. The pedestrian tried unsuccessfully to alert the driver about the box before the car sped away. The mailing label on the box indicated that, although the intended recipient of the box lived out of state, the sender of the box lived only a couple of blocks away. As required by state law with regard to lost property, the pedestrian decided to return the box to its sender. As the pedestrian reached the sender's residence, the pedestrian had a change of heart and decided to keep the unopened box and its contents for herself. Although the outside of the box gave no indication of its contents, there was a controlled substance inside the box. The applicable law provides that knowing possession of a controlled substance is a criminal offense. If the pedestrian were to be arrested before she arrived at her home with the unopened box, with which of the following offenses can she be charged? Answers: A: Both larceny and possession of a controlled substance B: Only larceny C: Only possession of a controlled substance D: Neither larceny nor possession of a controlled substance

Answer choice D is correct. Larceny is the trespassory taking and carrying away of the personal property of another with the intent to permanently deprive that person of the property. Under the continuing trespass rule, a trespass is deemed to be continuing when the defendant does not possess the necessary intent at the time of the taking but later develops the intent to permanently deprive the owner of the personal property. However, the initial taking of the personal property must have been wrongful. Here, the pedestrian's original taking of the box was not wrongful. State law required the pedestrian to return lost property to its owner, and that was her intent at the time she took possession of the box. Consequently, the taking was not wrongful, and the pedestrian has not committed larceny. Therefore, answer choices A and B are incorrect. Although the pedestrian clearly had possession of the controlled substance, the applicable law requires that her level of culpability must be "knowing." Because the box has not been opened, the pedestrian is not aware of the contents of the box and is therefore unaware that she possesses a controlled substance. Consequently, the pedestrian has not committed the offense of possession of a controlled substance. For these reasons, answer choices A and C are incorrect.

A landowner entered into valid contracts with both an artist and a builder to perform services on his property. Hoping to convert his property into a summer camp, he contracted with the builder to construct numerous quaint log cabins on his property, and he contracted with the artist to paint themed murals inside each cabin. Shortly after the builder began working on the property, he realized that the type of soil in the landowner's yard would cause the projected cost of the work to increase dramatically. After the landowner realized how high the cost of the cabins would be, he called the artist to tell her that he could not go through with their contract. The artist stated that she had already purchased paint and brushes for the job, and, inspired by the theme of each planned cabin, she had sculpted ceramic hanging wall plaques for the door of each cabin to match its theme. She had also rented a small utility van to transport her equipment due to the long distance to the landowner's rural property from her home. Which of the following will the artist be least likely to recover from the landowner? Answers: A: The contract price minus the market cost of performance B: The cost of the paint and brushes to create the murals C: The cost of the rented utility van to transport her equipment D: The cost of the sculpted hanging ceramic wall plaques

Answer choice D is correct. Nonbreaching parties may often choose between several types of damages, including reliance damages (those the plaintiff reasonably incurred in reliance upon the contract) and expectation damages (which put the injured party in the position that she would have been in if not for the breach). To calculate expectation damages, compare the value of performance without the breach (what was promised) with the value of the performance with the breach (what was received). Expectation damages must be foreseeable, and because the landowner and the artist contracted only for the artist to paint murals, the sculpted hanging ceramic wall plaques inspired by each cabin's theme would be an unforeseeable expense. Answer choice A is incorrect because it reflects expectation damages, which is one of the possible remedies for the nonbreaching party. Answer choice B is incorrect because the cost of supplies would likely constitute reliance damages. Answer choice C is incorrect because the artist incurred the cost in anticipation of completing work at the landowner's distant rural property, and thus she would be entitled to reimbursement for that expense incurred in reliance upon the contract.

Three states entered into a compact regarding the rights of each state to use water from a river that flowed through each state. The compact, which was approved by Congress, did not specify an enforcement mechanism. One of the states, contending that the other two states were violating the compact, filed suit in Supreme Court to enforce the compact. Pursuant to the U.S. Constitution, does the Supreme Court have jurisdiction to hear this action? Answers: A: No, because Congress approved the compact, so the dispute must be resolved by Congress. B: No, because the Supreme Court's jurisdiction is confined to appellate jurisdiction. C: Yes, because Congress has provided that the Supreme Court has original jurisdiction over disputes between two or more states. D: Yes, because the Supreme Court has original jurisdiction over all cases in which a state is a party.

Answer choice D is correct. Section 2 of Article III of the U.S. Constitution specifically provides that the Supreme Court has original jurisdiction over all cases in which a state is a party. Note that Congress cannot expand or limit this jurisdiction, but it can grant concurrent original jurisdiction to lower federal courts. Answer choice A is incorrect because, although an interstate compact may need the consent of Congress, the adjudication of controversies that arise with regard to an interstate compact is the province of the Supreme Court. Answer choice B is incorrect because, while the overwhelming majority of cases heard by the Supreme Court are a consequence of its appellate jurisdiction, the Supreme Court does have limited original jurisdiction pursuant to the Constitution. Answer choice C is incorrect because, while Congress can grant concurrent original jurisdiction to other federal courts, it cannot expand or limit the Supreme Court's original jurisdiction.

In response to unusual earthquake activity, a large city enacted an ordinance that required the owner of any habitable structure of more than two stories to permit the permanent placement of discrete and self-contained seismographic equipment owned by the city on the lowest floor of the structure. The purpose of this ordinance was to create an early detection network for seismic activity. The ordinance contained no provision for compensation of an owner of a structure subject to this ordinance because the space required for the equipment was minimal. Knowing that the city had enacted this regulation, a company purchased a tract of land and began construction of a three-story apartment building. The company sued, claiming that this ordinance violated its constitutional rights. Should the court rule in favor of the company? Answers: A: No, because the company knowingly acquired the rights to the property after the adoption of the challenged regulation. B: No, because the regulation has a minimal economic impact on the company. C: Yes, because the ordinance constitutes an impermissible exaction. D: Yes, because the ordinance results in a permanent physical occupation of the property.

Answer choice D is correct. The Due Process Clause of the Fourteenth Amendment incorporates the Takings Clause of the Fifth Amendment, thereby making it applicable to the states. A taking has occurred when the governmental regulation results in a permanent physical occupation of the property. In this case, the placement of the seismographic equipment in the building would result in the permanent physical occupation of the building by the city. Consequently, this placement would constitute a taking. Because the company would not be compensated for the taking, it is unconstitutional. Answer choice A is incorrect because a person who acquires property rights after the adoption of a regulation that affects those rights may nevertheless challenge the regulation as an unconstitutional taking. Answer choice B is incorrect because, even though the ordinance would have a minimal economic impact on the company because the equipment itself would occupy only a minimal space on the lowest floor of the apartment building, the ordinance mandates the permanent physical occupation of the property by the government's seismic-detection equipment. Answer choice C is incorrect because the ordinance is not an exaction. An exaction occurs when a local government exacts promises from a developer, such as setting aside a portion of the land being developed for a park, in exchange for issuing the necessary construction permits. Moreover, an exaction does not necessarily constitute an unconstitutional taking.

The owner of a building died. By the terms of his will, the building was devised to a religious organization "for so long as it is used for religious purposes." The rest of his real property interests were devised to his companion. By law, the owner's son was his only heir. Ten years later, the religious organization ceased to use the building for religious purposes. The religious organization executed a quitclaim deed of the building to a developer in exchange for valuable consideration. The developer plans to convert the building to commercial uses. The developer has brought an action to quiet title to the building against the companion, the son, and the religious organization. For whom should the court rule? Answers: A: The developer B: The religious organization C: The son D: The companion

Answer choice D is correct. The owner granted the religious organization a fee simple determinable in the building. A fee simple determinable is freely alienable, devisable, and descendible, but it is always subject to the stated condition. Upon the occurrence of the stated condition, the estate automatically reverts back to the grantor. The grantor's retained future interest is called a "possibility of reverter." A possibility of reverter is freely alienable by the grantor, both during his life and upon his death. Consequently, the owner properly devised this possibility of reverter to his companion. When the religious organization ceased to use the building for religious purposes, ownership of the building automatically reverted to the owner's companion. Answer choice A is incorrect because the developer purchased from the religious organization the rights that the organization had in the building. The religious organization's rights in the building ceased when the building was no longer used for religious purposes. For this reason, answer choice B is also incorrect. Answer choice C is incorrect because the owner of a possibility of reverter may transfer this interest by will, which, in this case, the owner of the building did.

A defendant was tried for theft of a motor vehicle and found guilty. As permitted by state law as a matter of right, he sought to appeal his conviction. His attorney, whom the defendant had hired, timely filed a notice of appeal. However, the appellate court dismissed the appeal due to his attorney's failure to file the necessary supporting documentation required by state law. The defendant subsequently challenged the action of the appellate court on the grounds of ineffective assistance of counsel. Will the defendant's challenge likely be successful? Answers: A: No, because the attorney timely filed a notice of appeal. B: No, because there is a presumption that an attorney's representation of a client was constitutionally adequate. C: No, because the protection against ineffective assistance of counsel only applies to state-provided counsel. D: Yes, because the attorney's unreasonable conduct prevented the defendant from pursuing his appeal.

Answer choice D is correct. There is a two-part test for establishing ineffective assistance of counsel: (i) the representation of a defendant by the defendant's attorney must fall below an objective standard of reasonableness, and (ii) the attorney's deficient performance prejudiced the defendant. Here, although the attorney took the necessary first step of filing a notice of appeal, the attorney unreasonably failed to take the second step of filing the necessary document. As a consequence, the defendant was prejudiced (i.e., his appeal was dismissed without a hearing). Answer choice A is incorrect because, although the defendant's attorney did timely file a notice of appeal, the attorney's failure to file the documentation necessary to pursue the appeal was unreasonable. Answer choice B is incorrect because, although there is a presumption that an attorney's representation of a client was constitutionally adequate, this presumption may be overcome. Answer choice C is incorrect because the protection against ineffective assistance of counsel extends to an attorney hired by a defendant as well as to court-appointed counsel.

In a civil assault suit between a plaintiff and a defendant, a witness testified that the defendant had been with her on the night of the alleged assault, more than 200 miles away from where the assault was alleged to have occurred. To challenge the witness's credibility, the plaintiff's attorney sought to present evidence of the witness's juvenile conviction for voluntary manslaughter five years ago. The defendant objected to the admission of this evidence. Should the judge admit the evidence? Answers: A: Yes. B: Yes, but only if the judge concludes that the probative value of admitting this evidence outweighs its prejudicial effect to the witness. C: No, but only if the judge concludes that the probative value of admitting this evidence is outweighed by its prejudicial effect to the witness. D: No.

Answer choice D is correct. Under FRE 609(d), evidence of a juvenile conviction is never admissible in a civil case. Accordingly, answer choices A and B are incorrect. Answer choice C is incorrect, as no determination of probative value is necessary. Note that, even in criminal cases, evidence of juvenile convictions has limited admissibility. Evidence of a juvenile conviction may be used to impeach a witness other than the defendant only if (i) it is offered in a criminal case, (ii) an adult's conviction for that same offense would be admissible to attack the adult's credibility, and (iii) admitting the evidence is necessary to fairly determine guilt or innocence.

A woman broke into her former lover's house at night with the intent to take back various items of her clothing that the former lover had refused to return to her. After conducting a search and being unable to find the clothing, she came across another woman's clothing. Extremely angry, she took a cigarette lighter and lit the lover's bed on fire, destroying the bed. With which of the following crimes can the woman be properly charged? Answers: A: Arson. B: Burglary. C: Both arson and burglary. D: Neither arson nor burglary.

Answer choice D is correct. Under the common law, arson is the malicious burning of the dwelling of another. Here, the woman burned only the former lover's bed, not an actual part of the structure of the dwelling. Accordingly, she cannot be guilty of arson and answer choices A and C are therefore incorrect. Answer choice B is incorrect. Common-law burglary is the breaking and entering of the dwelling of another in the nighttime with the specific intent to commit a felony therein. Here the woman did break and enter into the house of another, but not with the intent to commit a felony. Her intent was to recover her own clothing that was being wrongfully detained. Her decision to burn did not arise until after she entered the house, and would therefore not support a finding of burglary.

A borrower failed to make several payments due on a business loan. While the borrower was in the hospital recovering from a major operation, the lender, who was a family friend, approached the borrower's wife about additional security for the business loan. The lender intimated that, if some action was not taken with regard to the loan, the lender would have to file a civil action. Distraught over her husband's physical condition, the borrower's wife, at the lender's suggestion, granted the lender a mortgage on valuable property that she had recently inherited. The mortgaged property was to serve as additional security for the business loan. As a consequence of the mortgage, the lender did not reduce the outstanding amount of the business loan but did extend the time in which the borrower had to repay the loan. Of the following, which would be the best basis on which the borrower's wife can seek to avoid the mortgage? Answers: A: The borrower's wife granted the mortgage under duress. B: The borrower's wife lacked the capacity to enter into a contract. C: The lender did not give additional consideration for the mortgage. D: The mortgage was obtained through undue influence.

Answer choice D is correct. Undue influence is the unfair persuasion of a party to assent to a contract. Here, the circumstances surrounding the mortgage, such as the mortgagor's emotional trauma over her husband's physical condition, the friendship between the mortgagee and the mortgagor, the granting of the mortgage at the lender's suggestion, and the failure of the mortgagee to reduce the outstanding amount of the business loan despite the mortgaged property being valuable, all point to the mortgage being obtained through undue influence. While undue influence is not a certainty, it is the best available avenue for the wife to pursue rescission. Answer choice A is incorrect because, in order for a contract to be voidable for duress, there must be an improper threat that deprives a party of a meaningful choice. Generally, the threat of civil action is not improper unless it is made in bad faith. Here, the lender did not act in bad faith to compel the wife to grant the mortgage in a transaction unrelated to the loan, but instead sought the mortgage as additional security for the loan. Answer choice B is incorrect because emotional distress does not generally rise to the level of incapacity to contract. In order to challenge a contract on the grounds of mental incapacity, the party must have been unable to understand the nature and consequences of the transaction or unable to act in a reasonable manner with regard to the transaction and the other party had reason to know of this fact. Answer choice C is incorrect. Although the lender (mortgagee) had already made the loan to the borrower and the lender did not provide consideration for the mortgage by reducing the outstanding amount of the loan, the lender did extend the time for the payment of that loan. As a consequence, the lender did give additional consideration for mortgagor's granting the mortgage. Although this consideration directly benefited the mortgagor's husband rather than the mortgagor, a party's direct benefit from a contract need not be financial.

A mother and father had recently divorced, and the mother had remarried a man she met while the couple was separated. Although the mother's husband did not like children, the mother had fought for primary custody of the son she had with the father. The judge had awarded the mother primary custody and granted the father weekend visitations. The father was extremely angry with this result, believing that the mother fought for custody of the son solely to upset him. The father decided to get back at the mother. The father showed up at the mother's house alone when he was supposed to return the son from one of their weekend visits. When the mother and her husband answered the door, the father told the mother that the son had been in an accident and was in a coma. In fact, the son was playing happily at his grandmother's house. The mother and her husband, who each suffered severe emotional distress but no bodily harm as a consequence of the father's false report, have sued the father for intentional infliction of emotional distress. The father has moved to dismiss the husband's claim, arguing that his conduct was directed at the mother alone. Should the judge grant the father's motion to dismiss the husband's claim? Answers: A: No, because the husband is a member of the mother's immediate family. B: No, because the target of a defendant's conduct is irrelevant for such a claim. C: Yes, because the father intended to cause distress to the mother alone. D: Yes, because the husband did not sustain a physical injury.

Answer choice is A is correct. A defendant is liable for intentionally or recklessly acting with extreme and outrageous conduct, which conduct causes the plaintiff severe emotional distress. When the defendant's conduct is directed at a third-party victim, the defendant is liable if he intentionally or recklessly causes severe emotional distress to a member of the victim's immediate family who is present at the time of the defendant's conduct (and the defendant is aware of such presence), whether or not such distress results in bodily injury. In this case, the husband is a member of the mother's immediate family, he was present at the time of the father's conduct, and the father knew of his presence. Thus, the husband has the right to recover against the father (even if his claim will ultimately be unsuccessful). Answer choice B is incorrect because the target of a defendant's conduct is relevant. While claims for intentional infliction of emotional distress are usually only sustainable by the actual target of the extreme and outrageous conduct, and a third party can recover for such conduct only in limited circumstances, this is such a case. Answer choice C is incorrect because a member of the victim's family may recover under these circumstances, even though the conduct was directed at the mother. Answer choice D is incorrect because it is not necessary that the family member of a victim sustain a physical injury if the family member is present at the time of the conduct and the defendant is aware of the family member's presence.


Related study sets

pre-ch 11 Differential Analysis: The Key to Decision Making

View Set

Quiz 35-Linear Regression and r-squared

View Set

ATI Fundamentals Fundamentals Review 2019 *

View Set

Finance 401-Chapter 10, 11, 13 Homework #3

View Set

PHED 1164 FINAL EXAM Study Guide

View Set